Archer Neuro

Réussis tes devoirs et examens dès maintenant avec Quizwiz!

A client with Alzheimer's disease is being cared for by the nurse. Which nursing problem for the client would be the nurse's primary concern? A. Inability to do activities of daily living. B. Increased risk for injury. C. Potential for constipation. D. Ineffective family coping.

Explanation Choice A is incorrect. Alzheimer's patients have difficulty completing activities of daily living. However, the nurse should prioritize client safety over other problems. Choice B is correct. Safety should be the highest priority for the client. Clients with Alzheimer's disease are unaware of their surroundings and tend to wander. The nurse should implement safety measures. Choice C is incorrect. Older clients have an increased risk for constipation; however, their safety should take priority for other concerns. Choice D is incorrect. It is understandable for the family to be troubled regarding their loved one's condition. However, the nurse should always prioritize the client's safety over other problems.

The nurse is planning care for a client with homonymous hemianopia. The nurse should plan for which intervention in the care plan? A. Place an eye patch over the affected eye B. Instruct the client to turn their head from side to side C. Speak slowly, clearly, and in a deeper voice D. Provide the client with ear plugs to promote rest

Explanation Choice B is correct. Homonymous hemianopia (HH) is vision loss on the same side of the visual field in both eyes. It is appropriate for the nurse to teach the client to scan the room. Scanning the room will expand the visual field because the same half of each eye is affected. Choices A, C, and D are incorrect. An eye patch is an appropriate intervention for a client with double vision (diplopia). HH is not a problem with hearing and changing the approach to speaking to a client and providing ear plugs is irrelevant to this disorder. Additional Info Homonymous Hemianopia is characterized by vision loss on the same side of the visual field in both eyes. This is usually caused by a stroke, tumors, or epilepsy. Visual field loss is indicative of a lesion involving the visual pathway posterior to the chiasm.

What is the leading cause of cognitive impairment in old age? A. Stroke B. Malnutrition C. Alzheimer's disease D. Loss of cardiac reserve

Explanation Choice C is correct. Alzheimer's disease is the most common degenerative neurological illness and the most common cause of cognitive impairment. It is irreversible, progressing from deficits in memory and thinking skills to an inability to perform even the simplest of tasks. Choices A, B, and D are incorrect. NCSBN Client Need Topic: Physiological Integrity, Subtopic: Physiological Adaptation - Cognitive Changes in Aging Last Updated - 02, Feb 2022

The ICU nurse assesses a comatose patient with a known lesion to the medulla. Which breathing pattern would the nurse expect to assess? A. Cheyne-Stokes B. Apneustic breathing C. Central neurogenic hyperventilation D. Cluster breathing

Explanation Choice D is correct. Cluster breathing is associated with lesions of the medulla or lower pons. This breathing pattern is characterized by clusters of breaths with irregular pauses in between. Choice A is incorrect. Cheyne-Stokes is associated with bilateral hemispheric disease or metabolic brain dysfunction and commonly occurs at the end of life. This breathing pattern is associated with cycles of hyperventilation and apnea. Choice B is incorrect. Apneustic breathing is associated with lesions of the mid or lower pons. This breathing pattern is characterized by a prolonged inspiratory phase or pauses alternating with expiratory pauses. Choice C is incorrect. Central neurogenic hyperventilation is associated with lesions of the brainstem between the lower midbrain and upper pons. This breathing pattern is characterized by sustained, regular, rapid, and deep breathing.

A newly registered nurse is caring for a school-aged child with cerebral palsy under the supervision of a senior nurse. Which action by the new RN would warrant the senior nurse to intervene? A. The new RN initiates gentle range-of-motion exercises to the client. B. The new RN lowers the bed to its lowest position. C. The new RN wheels the client to the playroom via wheelchair. D. The new RN feeds the child with the bed elevated at 30 degrees.

Explanation Choice D is correct. The nurse should position the client with the head of the bed elevated at 60 - 90 degrees to prevent aspiration. Choice A is incorrect. ROM exercises prevent contractures in the child with cerebral palsy. Choice B is incorrect. Lowering the bed in its lowest position decreases the risk of injury during a fall. Choice C is incorrect. Taking the client to the playroom gives the client a chance to play. This action of the nurse represents patient advocacy. Last Updated - 14, Feb 2022

Which of the following statements are true regarding neural tube defects? Select all that apply. A. Types of neural tube defects include spina bifida occulta, spina bifida cystica, meningocele, and myelomeningocele. B. The nurse should protect the exposed sac by covering with a sterile, moist, non-adherent dressing. C. Left-lateral is the optimal position to minimize tension on the sac. D. Neurological deficits are always present in patients with neural tube defects.

Explanation Choices A and B are correct. Spina bifida occulta, spina bifida cystica, meningocele, and myelomeningocele are the types of neural tube defects (Choice A). If there is exposed spinal cord or meninges in a sac, it is essential to cover them with a sterile, moist, non-adherent dressing. This prevents infection and maintains the moisture of the pouch containing the spinal cord and meninges (Choice B). Choice C is incorrect. Prone is the best position to place the infant so that tension is minimized and the risk of trauma is reduced. Choice D is incorrect. Neurological deficits are not present with all neural tube defects. With spina bifida occulta and meningocele, neurological deficits are not usually present. NCSBN Client Need: Topic: Physiological Integrity, Subtopic: Physiological Adaptation, Neurology Last Updated - 04, Nov 2021

This nurse is caring for a client who is receiving prescribed carbamazepine. Which of the following findings would indicate a therapeutic response? A. Decreased mood lability B. Steady gait C. Urinary continence D. Increased bone mass

Explanation Choice A is correct. Carbamazepine is indicated for the prevention of seizures—neuropathic pain. And the treatment of certain mood disorders. The client demonstrating decreased mood lability would be the desired outcome. Choices B, C, and D are incorrect. Increased bone mass, urinary continence, and steady gait are all outcomes irrelevant to carbamazepine.

A client has been diagnosed with Parkinson's disease and was prescribed carbidopa-levodopa (Sinemet). Which objective assessment point would indicate that the treatment has been effective? A. The tremors have lessened in frequency B. The frequent swallowing has stopped C. The seizures have spaced out D. There is decreased lacrimation

Explanation Choice A is correct. The classic sign of Parkinson's disease is the "pill-rolling" tremors of the hands. Festinating gait is also present in the lower extremities. Treatment is considered valid when these tremors are lessened. Choices B, C, and D are incorrect. The condition does not involve increased swallowing and lacrimation. Seizures are also not associated with Parkinson's disease.

Which position is the most appropriate to prevent foot drop for a client who is on bed rest following a spinal injury? A. Supination B. Dorsiflexion C. Hyperextension D. Abduction

Explanation Choice B is correct. Dorsiflexion is the most appropriate position to prevent foot drop in a client on bed rest following a spinal injury. Choices A, C, and D are incorrect. None of these positions would be used to prevent foot drop. A: Supination involves lying clients on their back or facing a body part upward. B: Hyperextension is a state of exaggerated extension. D: Abduction involves the lateral movement of a body part away from the midline of the body.

The nurse is taking care of an 8-hour post-operative spinal surgery client. What should be the priority nursing intervention for the client? A. Assess how much opioid analgesics the client is using via the patient-controlled analgesia (PCA) pump. B. Logroll the client with three staff when turning the client from side to side. C. Assist the client in ambulating to the bathroom. D. Place pillows under the thighs of each leg when the client is in the supine position.

Explanation Choice B is correct. Logrolling the client is a priority to maintain proper body alignment and prevent injury to the spinal cord. Choice A is incorrect. The PCA delivers a fixed amount of analgesic to the client every time he presses the button. The priority of the nurse should be to prevent spinal cord injury to the client. Choice C is incorrect. The client is in the first 24 hours post-surgery and should be on bed rest. Ambulation is not a priority at this time. Choice D is incorrect. The client can put pillows under the client's legs to increase comfort; however, the priority nursing diagnosis is to prevent post-operative complications. Last Updated - 15, Oct 2021

The nurse is caring for a client prescribed tizanidine. The nurse understands that this medication has had a therapeutic effect when the client reports A. increased ability to focus. B. decreased muscle spasms. C. improved short-term memory. D. sleeping without awakening at night.

Explanation Choice B is correct. Tizanidine is a muscle relaxant and is utilized in the treatment of multiple sclerosis. Other indications for a muscle relaxant include an injury such as a motor vehicle crash that may cause muscle spasms. Choice A, C, and D are incorrect. Tizanidine is a muscle relaxer and a central nervous system depressant (CNS). It is highly unlikely this medication would increase the ability to focus as it causes drowsiness. The drowsiness is likely to cause memory impairments, especially if it is administered to an older adult. This medication is not indicated for insomnia. Additional Info Additional Information. Muscle relaxers are central nervous system depressants. Key teaching points for this medication (and others in this class) include avoiding driving while taking the medication, and not combining the medication with alcohol. Other medicines included in this class include baclofen, carisoprodol, cyclobenzaprine, and methocarbamol.

The nurse assesses an infant who sustained a traumatic brain injury (TBI). Which assessment finding requires follow-up? Select all that apply. A. Bulging fontanel B. Tachycardia C. Bradycardia D. Ptosis E. Distended scalp veins

Explanation Choices A, C, and E are correct. A tense, bulging fontanel is a classic sign of increased ICP in an infant. Associated symptoms that are concerning include bradycardia and distended scalp veins. Choices B and D are incorrect. Tachycardia is a clinical manifestation of shock but not for increased ICP. The client would exhibit triad symptoms such as bradycardia, bradypnea, and widening pulse pressure. Ptosis is drooping of the eyelid and is not associated with increased ICP. Pupillary changes would be assessed as a late sign of increased ICP, which would be nonreactive on an assessment. Additional Info For an infant with a TBI, the nurse must assess the newborn for increased intracranial pressure. Manifestations of increased ICP in newborns and infants include a high-pitched cry, bulging fontanels that may also have distended scalp veins, irritability, bradycardia, and an irregular breathing pattern. Last Updated - 16, Apr 2022

The nurse is reviewing laboratory data for a client with epilepsy taking prescribed valproic acid (VPA). The client's VPA level is 40 mcg/mL. Which action should the nurse take next? A. Evaluate the client for non-adherence. B. Instruct the client to skip the next scheduled dose. C. Assess the client for VPA toxicity. D. Document the result as within normal limits.

Explanation Choice A is correct. The therapeutic VPA level is 50-125 mcg/mL. A VPA level of 40 mcg/mL is considered sub-therapeutic and requires follow-up as the client is at risk of seizure. Choices B, C, and D are incorrect. Skipping the next dose of VPA, assessing for toxicity, and documenting the result as therapeutic/within normal limits would be inappropriate as a level of 40 falls outside of the therapeutic range. Additional Info VPA is indicated in preventing seizures, treatment for bipolar disorder, and migraine headache prevention. The most common adverse effects of VPA include nausea, vomiting, blood dyscrasias, hair loss, and metabolic syndrome. The liver enzymes should be monitored while a client takes VPA as hepatic injury may occur. Last Updated - 16, Feb 2022

Of the following, which conditions would the nurse recognize as potential sources of neuropathic pain? Select all that apply. A. Spinal tumor B. Arthritic joint C. Muscle strain D. Shingles E. Kidney stones

Explanation Choices A and D are correct. Neuropathic pain describes constant inflammation or irritation of nerve cells that causes pain sensation due to oversensitive nerve cells and a decrease in opioid receptors. Examples of neuropathic pain sources include CNS lesions, stroke, tumor, multiple sclerosis, sciatica, shingles, and phantom limb pain. Choices B, C, and E are incorrect. Nociceptive pain is typically predictable and temporary based on the injury. Examples of nociceptive pain sources include kidney stones, menstrual cramps, muscle strain, venipuncture, and arthritic joints. Last Updated - 08, Feb 2022

The nurse is developing a plan of care for a client with advanced Alzheimer's disease. Which of the following should the nurse include? Select all that apply. A. Assess the client's risk for falls B. Monitor the client for hyperorality C. Provide consistent caregivers D. Obtain a prescription for as-needed (PRN) diphenhydramine E. Foster a low-stimulation environment F. Offer limited choices Submit Answer

Explanation Choices A, B, C, E, and F are correct. Advanced Alzheimer's disease requires the nurse to utilize skill and patience. This progressive neurodegenerative disorder causes memory impairments but then erodes executive functioning, which may lead to indecisiveness, behavioral disturbances, impulsivity, and emotional lability. Assessing the client for falls is universal and should occur for a client with AD, especially with advancing AD, because of the client's inability to make sound judgments. Hyperolality is when the client puts everything in the mouth to taste and chew. As executive functioning declines, this may manifest and present a choking hazard. The nurse should ensure objects such as batteries, safety pins, buttons, etc., are not within the client's immediate environment. Consistent caregivers are always a good option (AD or not). Consistent caregivers decrease anxiety because it allows rapport building and promotes a sense of trust. For the client with AD, this familiarity may reduce the emotional lability associated with anxiety. A low-stimulation environment is helpful for the client with AD. Flashing lights and loud noises may create a sense of anxiety which can make the client react negatively. Keeping an environment that is predictable and calm would be helpful. Choices builds client autonomy, but too many choices could increase anxiety. The client with AD should be offered limited choices to promote decisiveness but not create doubt. For example, having the client decide between the an entree of pork or chicken versus pork, chicken or fish. Choice D is incorrect. Anticholinergics, such as diphenhydramine worsen memory impairments and should not be utilized. While they may be used to cause chemical sedation, the nurse should recommend against these medications. Clients with AD may also have a paradoxical effect with these medications where they become more activated instead of sedated when exposed to these agents. Additional Info ✓ Alzheimer's disease (AD) is the most common type of dementia. ✓ The progressive deterioration of cognitive functioning commonly characterizes AD. ✓ Initial deterioration may be so subtle and insidious that others may not notice it. ✓ In the early stages of the disease, the affected person may be able to compensate for and hide cognitive deficits. ✓ Manifestations of AD include impairment of recent memory which progresses to remote memory. ✓ As the disease progresses, the client may have impairment in executive functioning, including planning, organizing, and problem-solving. ✓ Additional clinical features of AD include the triple-A (agraphia, agnosia, and apraxia). ✓ Nursing care is aimed at maximizing function and promoting safety. The nurse should also provide caregiver/family support because of the emotional toll of this disorder. Last Updated - 11, Feb 2023

A lumbar puncture was performed on a client for a myelogram. After the procedure, the client complains of a severe headache. The most appropriate nursing intervention is: A. Increase the client's oral fluid intake B. Administer the prescribed antihypertensives to the client C. Give the client roll lenses D. Place a cool pack over the lumbar puncture site

Explanation Choice A is correct. A lumbar puncture (LP) reduces a client's cerebrospinal fluid volume and pressure. As a result of this decreased volume and pressure, headache results. This post-lumbar puncture headache is a common post-procedure complication, usually occurring hours to one to two days following the procedure, with severity ranging from moderate to severe. Hydration is a primary treatment for post-lumbar puncture headache. Increasing the client's fluid intake would facilitate the restoration of the client's cerebrospinal fluid volume. Choice B is incorrect. The administration of antihypertensive medications will not address the issue at hand. Currently, the client is experiencing a post-lumbar puncture headache due to decreased cerebrospinal fluid volume and pressure. Antihypertensive medications will not increase the volume or pressure of the cerebrospinal fluid. If the antihypertensive medications affect the cerebrospinal fluid, the medications will decrease the pressure, worsening the client's headache. Choice C is incorrect. Roll lenses act as a blindfold to reduce light irritation to the eyes for a client experiencing photophobia. Although this may benefit some clients, this is not the most appropriate nursing intervention. Choice D is incorrect. Administering antihypertensives will not address the problem. Placement of a cool pack on the puncture site may help temporarily reduce pain or inflammation, but this nursing intervention does not address the primary issue causing the client's headache. Therefore, this is not the most appropriate nursing intervention.

The nurse is caring for a client with Huntington's disease. Which of the following assessment findings would be expected? Select all that apply. A. Halitosis B. Chorea C. Hallucinations D. Hematemesis E. Weight loss

Explanation Choices B, C, and E are correct. The cardinal features of Huntington's disease include chorea (brief, involuntary movements involving the trunk, limbs, and face). This disease has common psychiatric symptoms such as depression, paranoia, delusions, and hallucinations. Weight loss is also a common finding caused by the excessive energy expended in abnormal movements. Choices A and D are incorrect. Halitosis and hematemesis are gastrointestinal manifestations associated with peptic ulcer disease. These are not associated with Huntington's disease. Additional Info Huntington's disease is a neurodegenerative disease that is not well understood. This disease causes neuropsychiatric symptoms such as chorea, dystonia, abnormal eye movements, hallucinations, and weight loss. Treatments include VMAT2 inhibitors such as tetrabenazine. Adjunctive treatments such as benzodiazepines may be utilized for symptoms such as uncontrolled chorea.

The nurse is preparing a staff in-service regarding conductive hearing loss. It would be appropriate for the nurse to identify which factors cause this type of hearing loss? Select all that apply. A. Presbycusis B. Prolonged exposure to noise C. Foreign body D. Ototoxic substance E. Cerumen

Explanation Choices C and E are correct. Conductive hearing loss is typically reversible and caused by cerumen, foreign body, tumor, edema, and acute infection. Choices A, B, and D are incorrect. These are all causes of sensorineural hearing loss, which is often irreversible. Additional Info Hearing loss is divided into sensorineural or conductive. Conductive hearing loss is caused by obstruction. Causes of this type of hearing loss include cerumen, foreign body, water, edema, infection, or tumor. This type of hearing loss may be reversible. Impairments of the nerve fibers cause sensorineural hearing loss. Causes of this type of hearing loss include prolonged exposure to noise, ototoxic substances (aminoglycosides), diabetes mellitus, and presbycusis (age-related hearing loss). This type of hearing loss is often not reversible.

You are working in the emergency department when a patient with a suspected stroke arrives. According to the American Heart Association (AHA), all of the tasks listed below should be done for this patient. What is the correct sequence for these tasks? Neurologic assessment by the stroke team General assessment and stabilization Determine if the patient is a candidate for fibrinolytic therapy Administer rtPA Submit Answer

Explanation Correct ordered sequence: According to the AHA's suspected stroke algorithm, the correct course for the treatment of the stroke patient is: General assessment and stabilization within 10 minutes of arrival to the ED Neurologic evaluation by the stroke team within 25 minutes of entry to the ED CT scan and determination if there is intracranial hemorrhage within 45 minutes of entry to the ED If ischemic stroke, determine if the patient is a candidate for fibrinolytic therapy using the fibrinolytic checklist Administer rtPA within 60 minutes of entry to the ED Admit to the stroke unit within 3 hours of entry to the ED NCSBN Client Need Topic: Management of Care, Sub-Topic: Establishing Priorities, Neurologic

You have just arrived for your shift and are assigned a 32-year-old client who arrived at your facility four days ago following a motor vehicle accident. The client sustained a traumatic head injury in the collision and was intubated in the field. Earlier today, the client was extubated. During your focused neurological assessment, you are required to calculate the client's Glasgow Coma Scale and, if required, anticipate any forthcoming interventions. Upon performing your assessment, you find the client opens his eyes once you begin speaking but fails to obey your commands. He does withdraw from pain but does not make any purposeful movements. Additionally, the client does not answer your questions. His responses seem somewhat incoherent, consisting primarily of incomprehensible sounds and mumbling. What is this client's Glasgow Coma Scale (GCS) score, and what would your next steps be? A. This client's GCS score is 6. Respiratory therapy needs to be contacted, and the client needs to be intubated immediately. B. This client's GCS score is 7. The provider needs to be contacted to obtain an order for intubation. Once the order is in place, respiratory therapy will be contacted, and the client will be intubated. C. This client's GCS score is 11. The client will continue to be monitored. D. This client's GCS score is 9. The client's current GCS score will be compared to prior GCS scores, and although the score is not currently <8, intubation equipment would be in or directly outside the room.

The Glasgow Coma Scale is a valuable tool for completing a rapid assessment of one's level of consciousness. The Glasgow Coma Scale score ranges from 3 to 15, scoring in three categories: eye opening, motor response, and verbal response. In terms of the Glasgow Coma Scale (GCS), remember the phrase: "Less than eight, intubate." Explanation The correct answer is D. Here, the client opens his eyes to voice (3), withdraws to pain (4), and makes incomprehensible sounds (2), therefore totaling a Glasgow Coma Scale (GCS) score of 9. Due to the score of 9, the nurse should first compare the calculated GSC score to the prior scores to ensure this is not a dramatic shift in the client's trajectory. For example, if the client was extubated earlier today with a documented GCS score of 11, then the health care provider should be contacted immediately. Regardless, with the GCS score of 9, the nurse should ensure that intubation equipment is in or directly outside the room and in working condition. Choice A is incorrect. This client's Glasgow Coma Scale score is 9, not 6. With a GCS score of 6, the client should currently be intubated. Moreover, depending on facility policy, often, other individuals are capable of intubating in a timelier manner (i.e., resident health care providers, fellows, rapid response nurses, etc.). Choice B is incorrect. This client's Glasgow Coma Scale score is 9, not 7. With a GCS score of 7, the client should currently be intubated. Moreover, depending on facility policy, there may be a standing order for intubation in emergencies, so you must be aware of the policies and procedures at every facility you staff. Additionally, frequently other individuals are capable of intubating in a timelier manner (i.e., resident health care providers, fellows, rapid response nurses, etc.). Choice C is incorrect. This client's Glasgow Coma Scale score is 9, not 11. Learning Objective Calculate the client's Glasgow Coma Scale (GSC) score and determine what nursing interventions should occur next.

The RN is caring for a client who is recovering from carotid endarterectomy. Which assessment would the nurse recognize as a sign that the client experienced hypoglossal nerve injury? A. Tongue deviation B. Inspiratory stridor C. Tracheal deviation D. Severe headache

Explanation Choice A is correct. Carotid endarterectomy is a procedure to remove plaque build-up in the carotid artery to improve blood flow to the brain. Complications following this procedure include cranial neuropathies, hemodynamic instability, stroke, myocardial infarction, and hematoma. Hypoglossal nerve damage is one of the most common cranial neuropathies following carotid endarterectomy and would be displayed as tongue deviation. Choice B is incorrect. Inspiratory stridor or hoarseness would suggest injury to the laryngeal nerve. Choice C is incorrect. A tracheal deviation would suggest possible carotid artery hemorrhage due to blood collecting and putting pressure on the airway. Choice D is incorrect. A severe headache following this procedure would suggest cerebral hyperperfusion syndrome (CHS) due to increased blood flow to the brain and insufficient cerebrovascular autoregulation.

The nurse is performing medication reconciliation for a client with Parkinson's disease. Which medication should the nurse question with the primary healthcare provider (PHCP)? A. Phenelzine B. Levodopa C. Pramipexole D. Ropinirole

Explanation Choice A is correct. Phenelzine, also known as Nardil, belongs to a class of medications known as MAOI inhibitors that are not safe for clients with Parkinson's disease and may precipitate a hypertensive crisis. A hypertensive crisis is a sudden increase in blood pressure, which may be life-threatening. Choices B, C, and D are incorrect. All of these medications are indicated for Parkinson's disease. Levodopa is a typical medication used to treat Parkinson's disease and is commonly combined with cabidopa. Pramipexole is a typical medication used to treat Parkinson's disease. Ropinirole is a dopamine agonist and is used to treat Parkinson's disease. Additional Info Source : Archer Review Parkinson's disease is a progressive neurological disorder that results in manifestations such as ataxia, dyskinesia, and cognitive impairments. An array of medications are used to treat this disorder which includes levodopa-carbidopa, ropinirole, diazepam, and pramipexole. Last Updated - 06, Jan 2022

The nurse is caring for a client who sustained a cervical spinal cord injury. Which priority vital sign should the nurse obtain? A. Respiratory rate B. Blood Pressure C. Pulse D. Temperature

Explanation Choice A is correct. Respiratory rate is essential to monitor when a cervical spinal cord injury is sustained. The upper cervical spinal nerves innervate the diaphragm to control breathing. Thus, specific injuries to the cervical spinal cord could be catastrophic. Choices B, C, and D are incorrect. These assessments are not a priority to monitor because the cervical spinal cord nerves innervate the diaphragm to control breathing. So these vital signs would not be directly relevant. Additional Info The upper cervical spinal nerves innervate the diaphragm to control breathing. Monitor all individuals with a spinal cord injury for respiratory problems, diaphragmatic breathing, and for diminished or absent reflexes in the airway (cough and gag). Until a cervical spinal cord injury can be excluded, the client should have immobilization via a cervical spinal cord collar. Last Updated - 31, Jul 2022

The nurse is caring for a client with narcolepsy. The nurse anticipates which prescription from the primary healthcare provider? A. Trazodone B. Modafinil C. Diazepam D. Fluoxetine

Explanation Choice B is correct. Narcolepsy is a disorder characterized when a client unexpectedly falls asleep in the middle of normal daily activities. Agents to keep the client awake during the day are the treatment goal. A common medication used is modafinil. Modafinil is a central nervous stimulant dosed during daylight hours to keep the client alert. Choices A, C, and D are incorrect. The treatment goal for narcolepsy is for the client to stay awake during the day. Trazodone and diazepam are central nervous system depressants and would cause sleepiness. Fluoxetine is not indicated in the treatment of narcolepsy as this medication modulates serotonin and is helpful for anxiety and depressive disorders. Additional Info Narcolepsy is a condition characterized by the client experiencing 'sleep attacks,' which may cause serious injury if one should occur while the client is driving, etc. The client with narcolepsy may also have Cataplexy, a sudden skeletal muscle weakness. The condition is often associated with strong emotions (e.g., joy, anger), and commonly the knees buckle, and the individual falls to the floor while still awake.

Which of the following is the reason a patient receives nitrous oxide in addition to thiopental sodium? A. To provide the additional anesthesia to put him in a sleep-like state B. To increase the effectiveness of each drug at lower dosages C. Because thiopental sodium is not effective when used alone D. Because nitrous oxide is not effective when used alone

Explanation Choice B is correct. Nitrous oxide may be used for dental procedures or brief obstetrical or surgical procedures. It may also be used together with other general anesthetics, which makes it possible to decrease the dosages of each with greater effectiveness. There are two primary methods of causing general anesthesia. IV agents are usually administered first because they act within a few seconds. After the patient loses consciousness, inhaled agents are used to maintain the anesthesia. Choice A is incorrect. Nitrous Oxide has low potency and does not produce complete loss of consciousness or extreme relaxation of skeletal muscles. It does not cause surgical anesthesia, so it is commonly combined with other curative anesthetic agents. Choice C is incorrect. Pentothal can be used alone. It has an onset time of fewer than 30 seconds and a duration of only 10-30 minutes. Therefore, it is used for brief medical procedures or to rapidly cause unconsciousness before administering inhaled anesthetics. Choice D is incorrect. Nitrous oxide can be used alone for simple procedures, such as dental procedures. It allows the patient to remain conscious and follow instructions while experiencing full anesthesia. NCSBN Client Need Topic: Physiological Integrity, Subtopic: Pharmacological Therapies Last Updated - 24, Dec 2021

The nurse is caring for a client newly prescribed ropinirole. The nurse understands that this medication is prescribed to treat which condition? A. Multiple Sclerosis B. Parkinson disease C. Schizophrenia D. Guillain-barré syndrome

Explanation Choice B is correct. Ropinirole is a dopaminergic drug used in conjunction with other medications to treat Parkinson's disease. Additionally, this medication is indicated to treat restless leg syndrome. Choices A, C, and D are incorrect. Multiple sclerosis is treated by interferons, muscle relaxers, and steroids. This medication would be detrimental for an individual with schizophrenia because it increases dopamine levels which can trigger psychosis. Guillain-barré syndrome is treated with plasmapheresis or immune globin. Ropinirole is used along with other medications to help mitigate the symptoms of Parkinson's disease. This medication is also approved to treat symptoms of restless leg syndrome. Common adverse effects include agitation, delirium, peripheral edema, and gi upset.

The nurse is caring for a client receiving prescribed sumatriptan. Which client report would indicate that the client is experiencing an adverse response? A. Nervousness B. Warm sensation C. Angina D. Tingling sensation

Explanation Choice C is correct. Angina is a concerning finding and requires follow-up by the nurse. Vasoconstriction may occur with this medication, and thus, the client with a medical history of coronary artery disease, uncontrolled hypertension, and a previous stroke should not take this medication. Choices A, B, and D are incorrect. These are common (and expected) reactions to this medication. Thus, the nurse should educate the client to expect these sensations; the more they take the drug, the less they will experience these manifestations. Additional Info Sumatriptan is a medication indicated to abort migraine headaches. It is not a prophylactic treatment. Sumatriptan is commonly administered intranasally or subcutaneously. The nurse should educate the client that if they receive this medication as an injection, a warm and tingling sensation is normal and temporary. Contraindicated for clients with uncontrolled hypertension, coronary artery disease, and those who have sustained a stroke or myocardial infarction. The nurse should monitor the client's blood pressure, autonomic instability, altered mental status, and manifestations similar to serotonin syndrome. Common side effects include flushing, tingling, warmth, dizziness, gastric upset, nausea, and vomiting.

A nurse is caring for a newborn with a myelomeningocele. Prior to surgery, the most appropriate intervention to keep the sac sterile and protected is to: A. Leave the sac exposed to air. B. Apply petroleum jelly or ointment as a protective covering for the sac. C. Cover the sac with moist, saline dressings. D. Apply a dry dressing over the sac.

Explanation Choice C is correct. Prior to surgical closure, the sac is kept from drying by the application of a sterile, moist, nonadherent dressing. The moistening solution is usually sterile normal saline. Dressings are changed frequently (every 2 to 4 hours), and the sac is closely inspected for leaks, abrasions, irritation, and signs of infection. Choice A is incorrect. Exposing the sac to open air will result in a dehydrating effect on the sac. This is especially true when an overhead warmer is utilized due to the radiant heat. Choice B is incorrect. Prolonged use of petroleum jelly or ointments can result in the breakdown of the sac tissue. Choice D is incorrect. Dry dressings are irritating to the tissues of the myelomeningocele sac. Learning Objective When caring for a newborn with a myelomeningocele prior to surgical intervention, recognize the need to apply a sterile, moist, nonadherent dressing to the sac. Additional Info Myelomeningocele develops during the first 28 days of pregnancy when the neural tube fails to close and fuse. The largest number (75%) of myelomeningoceles occur in the lumbar or lumbosacral area. The location and magnitude of the defect determine the nature and extent of neurologic impairment. Meningoceles are repaired early, especially if the sac is in danger of rupturing. Last Updated - 22, Oct 2022

The nurse is caring for a client with newly prescribed sumatriptan. The nurse understands that this medication is intended to treat which condition? A. Peripheral artery disease B. Accelerated hypertension C. Migraine headache D. Angina

Explanation Choice C is correct. Sumatriptan is a 5-hydroxytryptamine (5-HT))-receptor agonist indicated for abortive migraine headache treatment. Sumatriptan stops headaches after they have begun ( abortive therapy), but it does not prevent them. Choices A, B, and D are incorrect. These conditions are all contraindications to the administration of this medication. Sumatriptan has been linked to the development of sudden myocardial infarction following its administration in at-risk individuals. Sumatriptan may induce vasoconstriction. A client with severe hypertension, coronary artery disease, history of stroke, and peripheral vascular disease should not take this medication. Learning Objective Recognize that 5HT receptor agonists such as Sumatriptan are used in abortive therapy of migraine headaches. Additional Info Sumatriptan is a medication indicated to abort migraine headaches. It is not a prophylactic treatment. Sumatriptan is commonly administered intranasally or subcutaneously. The nurse should educate the client that if they receive this medication as an injection, a warm and tingling sensation is normal and temporary. Contraindicated for clients with uncontrolled hypertension, coronary artery disease, and those who have sustained a stroke or myocardial infarction. The nurse should monitor the client's blood pressure, autonomic instability, altered mental status, and manifestations similar to serotonin syndrome. Common side effects include flushing, tingling, warmth, dizziness, gastric upset, nausea, and vomiting.

Your client has just got an epidural catheter to manage their severe, continuous pain. Which of the following is a nursing intervention that is necessary after the placement of this epidural catheter and the initiation of an opioid epidural infusion? A. Ensure the availability and immediate accessibility of Actiq to reverse any respiratory depression. B. Ensure the availability and immediate accessibility of Sublimaze to reverse any respiratory depression. C. Monitor the client at least every hour for the first 24 hours for any signs of respiratory depression and level of sedation. D. Monitor the client at least every 2 hours for the first 24 hours for any signs of respiratory depression and level of sedation.

Explanation Choice C is correct. The nursing intervention that is necessary after the placement of this epidural catheter and the initiation of an opioid epidural infusion is to monitor the client for signs of respiratory depression and level of sedation at least every hour for the first 24 hours. Choice A is incorrect. Ensuring the availability and immediate accessibility of Actiq to reverse any respiratory depression is not an appropriate nursing intervention. Actiq is a short-acting opiate that is used to treat breakthrough pain and will only increase respiratory depression, not reverse it. Choice B is incorrect. Ensuring the availability and immediate accessibility of Sublimaze to reverse any respiratory depression is not an appropriate nursing intervention. Sublimaze is an opioid medication and will only increase respiratory depression, not reverse it. Choice D is incorrect. Although the nurse must monitor the client carefully, this monitoring of the client for any signs of respiratory depression and level of sedation is inaccurate. Last Updated - 22, Oct 2021

Which of the following images shows the correct location to assess for Macewen's sign? Correct

Explanation Choice D is correct. Macewen's sign is a sign used to detect hydrocephalus. The examiner percusses on the skull at the junction of the frontal, temporal, and parietal bones and can auscultate a "cracked pot", or hyper-resonant sound if hydrocephalus is present. Macewen's sign is unrelated to congestive heart failure or chronic hypoxia. Choice A is incorrect. This is not the location to assess for Macewen's sign. This is the junction of the frontal and sagittal sutures. To assess for Macewen's sign, percuss the junction of the frontal, temporal, and parietal bones. Choice B is incorrect. This is not the location to assess for Macewen's sign. This is the junction of the parietal, temporal, and occipital bones. To assess for Macewen's sign, percuss the junction of the frontal, temporal, and parietal bones. Choice C is incorrect. This is not the location to assess for Macewen's sign. This is the junction of the sagittal and lambdoid sutures. To assess for Macewen's sign, percuss the junction of the frontal, temporal, and parietal bones. NCSBN Client Need: Topic: Psychosocial Integrity; Subtopic: Pediatrics - Neurology Last Updated - 13, Feb 2022

The nurse is caring for a client diagnosed with Multiple Sclerosis (MS). The nurse should anticipate a prescription for which medication? A. Topiramate B. Risperidone C. Prazosin D. Baclofen

Explanation Choice D is correct. Multiple Sclerosis (MS) may produce symptoms such as muscle spasticity, optic neuritis, fatigue, heat intolerance, and symptoms that seem to intensify on occasion (relapses). Muscle spasticity is best controlled with muscle relaxers such as baclofen. Choices A, B, and C are incorrect. Topiramate is an anticonvulsant drug indicated in the treatment of epilepsy as well as psychiatric conditions such as bipolar disorder. Risperidone is indicated for psychotic disorders such as schizophrenia. Prazosin is an antihypertensive that may be used for high blood pressure. This medication also may be indicated for psychiatric illnesses such as PTSD. MS symptoms are often vague and non-specific. These symptoms often include fatigue, optic neuritis, muscle spasms, heat intolerance, and difficulty with coordinating movements. Symptoms may exacerbate and remit. Treatment includes symptomatic care and interferon therapy.

The primary healthcare provider (PHCP) is preparing to intubate a client. The PHCP prescribes succinylcholine. The nurse understands that this medication is intended to A. sedate the client during the procedure. B. decrease oral and airway secretions. C. increase heart rate in case of a vagal response. D. cause skeletal and smooth muscle paralysis.

Explanation Choice D is correct. Skeletal and smooth muscle paralysis is the intent of this medication. Succinylcholine is a neuromuscular blocking medication typically given immediately prior to intubation to assist with the procedure. Choice A, B, and C are incorrect. Sedation does not occur with succinylcholine as it causes paralysis. This medication may be used adjunctively with sedatives for a client receiving mechanical ventilation. This medication does have an anticholinergic effect, but it is not given to decrease oral and airway secretions or prevent a vagal response. A medication that may be used to accomplish both would be atropine. Additional Info The muscle paralysis induced by depolarizing NMBDs (e.g., succinylcholine) is sometimes preceded by muscle spasms, which may damage muscles. These muscle spasms cause the release of potassium which may lead to hyperkalemia. Prolonged exposure to this medication may lead to hyperkalemia, and this medication should not be used if the client already has hyperkalemia. Finally, this medication may cause malignant hyperthermia. If a client develops a significant fever, muscle rigidity, and tachycardia, immediate treatment must be implemented. Last Updated - 18, Jul 2022

The nurse is caring for a client with a migraine headache. Which assessment findings should the nurse expect? Select all that apply. A. Unilateral frontotemporal pain B. Nausea C. Photophobia D. Fever E. Nuchal rigidity F. Vomiting

Explanation Choices A, B, C, and F are correct. An array of symptoms may be reported for a client experiencing a migraine headache (MH). The most common manifestations associated with an acute migraine headache include Unilateral frontotemporal pain that may be described as throbbing or dull Sensitivity to light (photophobia) and sound (phonophobia) Nausea and/or vomiting Altered mentation (drowsiness) Dizziness, numbness, and tingling sensations Choices D and E are incorrect. An acute migraine headache is not an infectious process. Thus, a fever and nuchal rigidity are not associated with this syndrome. If a client has a fever, nuchal rigidity, and photophobia, those findings are highly concerning for bacterial meningitis. Additional Info Migraine headaches have complex pathophysiology that is not entirely understood. The current thought process regarding this syndrome is that it is caused by a combination of neuronal hyperexcitability and vascular, genetic, hormonal, and environmental factors. During an acute migraine headache, often the client may feel as though they are experiencing a stroke because of transient facial paralysis and/or numbness that may be experienced. Last Updated - 18, Jul 2022

The nurse is preparing a staff in-service regarding sensorineural hearing loss. It would be appropriate for the nurse to identify which factors cause this type of hearing loss? Select all that apply. A. Diabetes mellitus B. Menieres disease C. Excessive cerumen D. Exposure to loud noise E. Excessive fluid

Explanation Choices A, B, and D are correct. These are all risk factors for sensorineural hearing loss. Diabetes may cause an insult to vasculature supplying the cochlea. Thus, causing hearing loss. Meniere's disease is a condition that features vertigo, hearing loss, and tinnitus. Exposure to loud noise is a significant risk factor because of the insult it causes to the nerve fibers. Choices C and E are incorrect. Obstruction in the ear is a cause of conductive hearing loss, which may be reversed. Additional Info Hearing loss is divided into sensorineural or conductive. Conductive hearing loss is caused by obstruction. Causes of this type of hearing loss include cerumen, foreign body, water, edema, infection, or tumor. This type of hearing loss may be reversible. Impairments of the nerve fibers cause sensorineural hearing loss. Causes of this type of hearing loss include prolonged exposure to noise, ototoxic substances (aminoglycosides), diabetes mellitus, and presbycusis (age-related hearing loss). This type of hearing loss is often not reversible. Last Updated - 11, Aug 2022

Which of the following examples describe chronic pain? Select all that apply. A. A patient is receiving chemotherapy for bladder cancer. B. An adolescent is admitted to the hospital for an appendectomy. C. A patient is experiencing a ruptured aneurysm. D. A patient who has fibromyalgia requests pain medication. E. A patient has back pain related to an accident that occured a year ago. F. A patient is experiencing pain from second-degree burns.

Explanation Choices A, D, and E are correct. Chronic pain is pain that may be limited, intermittent, or persistent that lasts beyond the average healing period. Examples of chronic pain include pain that is related to cancer, injuries (especially those that involve the nerves), and fibromyalgia. Acute pain is generally rapid in onset and varies in intensity from mild to severe, as occurs with an emergency appendectomy, a ruptured aneurysm, and suffering from burns. Choices B, C, and F are incorrect. These answer choices are examples of acute pain, not chronic. NCSBN Client Need Topic: Physiological Integrity, Subtopic: Basic Care and Comfort; Chronic Pain Last Updated - 09, Nov 2021

Select the sensory impairment that is accurately paired with one of its possible causes or a method for assessing it. Select all that apply: A. Impaired gustatory sensation: Using the Grady Scale B. Impaired tactile sensation: Diabetes C. Impaired auditory sensation: Using the Braden Scale D. Impaired Stereognosia: Alzheimer's disease E. Impaired Proprioception: Morse Scale

Explanation Choices B and D are correct. Impaired tactile sensation is often caused by peripheral neuropathy secondary to diabetes. Peripheral neuropathy, a long-term complication of diabetes, is characterized by the person's inability to feel things like heat, cold, and a painful stimulus like the prick of a needle in their feet. Impaired stereognosis is the lack of the client's ability to identify an everyday object with tactile sensations and without visual cues. Impaired stereognosis is associated with Alzheimer's disease. Choice A is incorrect. Grady Scales are used to determine levels of consciousness and not gustatory sensation. The impaired gustatory sensation is assessed by providing the client with small tastes of sweet, sour, salty, and spicy foods to identify their feelings. Choice C is incorrect. The impaired auditory sensation is assessed using an audiometer or a tuning fork. The Braden Scale is used to screen clients for their risk of developing a pressure ulcer. The Braden Scale uses scores from less than or equal to 9 to as high as 23. The lower the number, the higher the risk of developing a pressure ulcer. Score categories include 19-23 = no risk; 15-18 = mild risk; 13-14 = moderate risk or less than 9 = severe risk Choice E is incorrect. Proprioception is the sense of the relative position of one body segment to other body segments. Examples of tests to assess proprioception include the Finger-Nose test, the Heel-shin test, and the Thumb finding test. Morse scale assesses a patient's risk of falling, not proprioception. It consists of six variables that are quick and easy to score: History of falling - immediate or within three months; Secondary diagnosis; Ambulatory aids; Intravenous therapy; Gait and Mental status. Last Updated - 04, Dec 2022

Seizure precautions have been ordered for a patient admitted to the psychiatric unit. Which of the following nursing interventions is not appropriate when initiating seizure precautions? Select all that apply. A. Pad the side rails of the bed B. Lower side rails while the patient sleeps C. Remove hard or sharp objects from the bed D. Use four point restraints to prevent injury E. Adhere a fall risk bracelet to the seizure prone patient

Explanation Choices B and D are correct. Lowering the side rails and using four point restraints are not appropriate actions while deploying seizure precautions. Padded bed rails should remain up while the patient sleeps. Patients should be provided with a call light so that they may call for help if needed. Four-point restraints are not appropriate for the seizing patient and could result in injury. Choice A, C, and E are incorrect. These are appropriate seizure precautions. When initiating seizure precautions, the nurse should ensure that the side rails are padded ( Choice A). All sharp objects should be removed from a patient's bed when instituting seizure precautions ( Choice C). Patients prone to seizures should wear a fall risk bracelet to alert members of the health care team to the patient's need for increased supervision (Choice E). NCSBN client need Topic: Physiologic integrity, reduction of risk potential Last Updated - 25, Jan 2022

The nurse is caring for a client experiencing a tonic-clonic seizure. Which of the following medications should the nurse be prepared to administer? A. Lorazepam B. Phenytoin C. Carbamazepine D. Benztropine

Explanation Choice A is correct. A tonic-clonic seizure requires a client to be placed on their side and have their clothing loosened. Prompt intervention with benzodiazepines. In this case, prescribed lorazepam is given to break the seizure. Choices B and C are incorrect. Phenytoin and carbamazepine are anti-convulsants that are used to prevent seizures. Choice D is incorrect. Benztropine is an anticholinergic drug that is indicated for Parkinson's disease.

A 28-year old woman presents to the trauma bay after being shot in the upper back. She can move the left side of her body but is unable to move the right. However, she cannot feel any pain on the left. The nurse knows these symptoms are suggestive of which type of spinal cord injury? A. Incomplete spinal cord injury, central cord syndrome B. Incomplete spinal cord injury, Brown-Sequard syndrome C. Complete spinal cord injury, paraplegia D. Complete spinal cord injury, anterior cord syndrome

Explanation Choice B is correct. Based on the patient's symptoms of weakness on one side but sensory loss (pain sensation loss) on the other side, this is the best answer. Brown-Sequard syndrome is an incomplete spinal cord injury characterized as a weakness/paralysis (hemi-paraplegia) on the ipsilateral (same) side of the body and sensory loss (hemianesthesia) on the contralateral (opposite) side of the body below the level of injury. It is also known as the hemi-section of the spinal cord. At the level of the injury, there is complete loss of sensation and flaccid paralysis. Below the level of the injury, there is spastic paralysis and Babinski reflex (extensor plantar response) on the ipsilateral side. Brown-Sequard Syndrome may be due to traumatic or non-traumatic injuries. However, traumatic injuries such as gunshot wounds, stab wounds, motor vehicle accidents, or blunt trauma are more common causes than the non-traumatic etiologies. Spinal cord injuries (SCI) are classified as incomplete and complete. In an incomplete SCI, there is a partial loss of motor function and/or sensations below the level of injury. The client may exhibit more functioning/sensations in one extremity more than the other or on one side of the body than the other. Incomplete SCI also exhibits "sacral sparing" - some degree of preservation of sensation or motor function in the areas innervated by S4 to S5. In a complete SCI, there is a total loss of motor function and sensations below the level of injury. Both sides of the body are equally affected. Choice A is incorrect. Central cord syndrome is the most common form of incomplete SCI characterized by an inability to move the hands and arms (greater neurologic deficit in upper extremities than in lower extremities). Pain, temperature, light touch, and pressure sensation below the level of injury are lost. The spinal tracts that serve the arms are involved due to their central location in the cord, while the corticospinal tracts that supply the legs are spared due to their peripheral location. Most of the time, it is caused by trauma to the cervical spinal cord due to hyperextension in older patients with spinal stenosis/cervical degenerative joint disease or from flexion injuries in young patients. Choice C is incorrect. Complete Paraplegia is characterized as the inability to move the bottom half of the body and is often due to a complete spinal cord injury. Choice D is incorrect. Anterior cord syndrome is an incomplete SCI characterized by loss of pain/temperature sensation and motor function (paralysis) below the level of injury. This is due to injury to the anterior portion of the spinal cord or a decrease in the blood supply from the anterior spinal artery. Common causes are vertebral fractures, dislocations, and disc herniations. Spinothalamic tracts (pain/temperature sensation) and corticospinal fibers (motor function) in the anterior position of the cord are damaged. Touch and proprioception (position sensation) are preserved because those are controlled by dorsal (posterior) columns. NCSBN Client Need Topic: Physiological Adaptation; Sub-topic: Alterations in Body Systems Last Updated - 26, Dec 2021

The nurse is caring for a client immediately following the administration of prescribed sumatriptan. Which clinical finding would require follow-up by the nurse? A. Blood Pressure 189/98 mm Hg B. Headache pain 4/10 C. Client's skin appears flushed D. Reports of nausea

Choice A is correct. This significantly elevated blood pressure and requires follow-up. Vasoconstriction may occur with this medication, and thus, the client with a medical history of coronary artery disease, uncontrolled hypertension, and a previous stroke should not take this medication. Choices B, C, and D are incorrect. The medication is indicated for migraine headaches, and a headache would not be of significant concern. Flushing of the skin immediately after administering this medication is a common effect. Nausea and abdominal cramping may be a side-effect associated with this medication. Additional Info Sumatriptan is an abortive treatment for migraine headaches. This medication may be given intranasally or subcutaneously. The drug commonly causes flushing after administration and is expected. Recent myocardial infarction, peripheral vascular disease, and/or uncontrolled hypertension are contraindications to administering this medication. Last Updated - 02, Dec 2022

The nurse is educating a patient who is taking phenytoin. To make sure phenytoin does not fail, which over-the-counter (OTC) medication should the nurse advise the patient not to take at the same time? A. Acetaminophen B. Ibuprofen C. Calcium carbonate D. Ranitidine

Choice C is correct. Calcium carbonate (Tums) should not be taken at the same time as Phenytoin because taking them together can decrease the effects of phenytoin. Antacids containing calcium carbonate reduce the bioavailability of phenytoin by reducing both the rate of absorption and the amount of intake. Phenytoin is an anticonvulsant and not getting it at a therapeutic dose may result in the client having a recurrent seizure. Clients should be cautioned against the concomitant use of antacids/tums and phenytoin. If the client needs calcium carbonate, he should be instructed to separate the times of intake of calcium carbonate and phenytoin by at least two to three hours. Choice A is incorrect. Acetaminophen and phenytoin can be taken together without any concern for therapeutic failure. Choice B is incorrect. Ibuprofen and phenytoin can be taken together and do not cause the therapeutic failure of phenytoin. Choice D is incorrect. Ranitidine and phenytoin can be taken together and do not cause the therapeutic failure of phenytoin. Ranitidine may, however, increase the effects of Phenytoin, so the patient should be monitored for any phenytoin-related adverse effects.

Select the pain or pain management term that is accurately paired with its definition. A. Sensitization: The eradication of all pain sensation subsequent to the application of a repeated noxious stimulus. B. Allodynia: A sudden and spontaneous or illicited abnormal response that is unpleasant. C. Dysesthesia: A painful response to a nonpainful stimulus. D. Intractable pain: Severe and unyielding pain that cannot be corrected with medical treatments.

Choice D is correct. Intractable pain is severe and unyielding pain that cannot be corrected with medical treatments for the underlying cause. Pain management, rather than medical treatments, is indicated for intractable pain. Other commonly used pain management terms include: Sensitization: Heightened pain and increased sensitivity to a receptor after the application of a repeated noxious stimulus. Allodynia: A painful response to a nonpainful stimulus. Dysesthesia: A sudden and spontaneous or elicited abnormal response that is unpleasant. Choice A is incorrect. Sensitization is not the eradication of all pain sensation after the application of a repeated noxious stimulus. Sensitization is heightened pain and increased sensitivity to a receptor after the use of a repeated noxious stimulus. Choice B is incorrect. Allodynia is not a sudden and spontaneous or elicited abnormal response that is unpleasant. Allodynia is a painful response to a nonpainful stimulus. Choice C is incorrect. Dysesthesia is not a painful response to a nonpainful stimulus. Dysesthesia is an immediate and spontaneous or elicited abnormal response that is unpleasant.

Which of the following information would NOT be included in a client's pain history? A. The client's affective responses to pain B. The client's past alleviating measures C. The client's current vital signs D. The client's meaning of pain

Explanation Choice C is correct. The client's current vital signs would NOT be included in a client's pain history. However, these vital signs are part of the initial nursing assessment and ongoing assessments. Choice A is incorrect. The client's affective responses to pain are an integral part of a client's pain history; some emotional responses to pain include the client's feelings such as depression and anxiety in response to pain. Choice B is incorrect. The client's past alleviating measures that lessened their pain are an integral part of a client's pain history; therefore, this would be included in the client's pain history. Choice D is incorrect. The client's meaning of pain is an integral part of a client's pain history; therefore, this would be included in the client's pain history.

The nurse is evaluating the progress of a completely paraplegic female client with a C6-C7 spinal cord injury. Which indicator signifies that the client is improving in physical therapy? A. The client can control the motorized wheelchair. B. The client states she wants to stand up with assistance. C. The client says she wants to move her toes. D. The client says she regained her bladder control.

Explanation Choice A is correct. A C6-C7 spinal cord injury (SCI) can still retain some ability to extend shoulder, arms, and fingers with compromised dexterity in the hands and fingers. The client showing that she can maneuver a wheelchair indicates that she has progressed in therapy. Rehabilitation often will focus on learning to use the non-paralyzed portions of the body to regain varying levels of autonomy. Upon successful treatment, survivors of injuries at the C6/C7 level may be able to drive a modified car with hand controls. The C6 and C7 cervical vertebrae (and the C8 spinal nerve) form the lowest levels of the cervical spine and directly impact the arm and hand muscles. The C6/C7 injury has the potential to change everything below the top of the ribcage, resulting in quadriplegia or paraplegia. Physical therapy is an essential part of recovery. The patient will need to maintain any function not lost by the cord damage, as well as try to regain function. In acute rehabilitation of C6/C7 SCI patients, the focus is on strengthening the upper extremities to the maximal level in patients with complete paraplegia. Empowering exercises for shoulder rotation are proposed for using crutches, swimming, electric bicycles, and walking. At the end of the acute phase, strong upper extremities are needed for the independent transfer from the bed. For this purpose, active and resistance exercises to strengthen the muscles of the upper extremity should be initiated at the earliest possible period. The wheelchair is an essential tool for SCI patients to be mobile and participate in social life. Choices B and C incorrect. A client with C6-C7 Spinal cord injury with complete paraplegia loses control over leg and foot movement completely. These statements by the client indicate that she may need counseling regarding coping with her injuries. Choice D is incorrect. The client who loses control of their bladder may regain function again sometime after injury. However, this is not dependent on physical therapy and does not indicate a positive response to physical therapy.

The nurse is caring for a client with a spinal cord injury. Which client finding would require immediate follow-up? A. Absent bowel sounds B. Blood pressure 134/82 C. Pulse 92/minute D. Hyperreflexia

Explanation Choice A is correct. Absent bowel sounds, gastric distention, bradycardia, hypotension, and flaccid paralysis are concerning findings for spinal shock. When caring for a client following a spinal cord injury, spinal shock is one of the many complications which may occur within 48 hours following the injury. Choices B, C, and D are incorrect. This clinical data is not consistent with spinal shock. If spinal shock is suspected, the client will develop hypotension and bradycardia. This shock would depress reflexes, not cause hyperreflexia. Additional Info Spinal shock may occur immediately following a spinal cord injury or within 48 hours of the insult. The cause is thought to be the excessive amount of potassium in the extracellular space that reduces neural transmission. Manifestations associated with spinal shock include flaccid paralysis, absent bowel and bladder control, and loss of reflex activity. Males may develop priapism. Treatment involves frequent assessment of vital signs, correcting any fluid or electrolyte abnormalities, and prompt administering corticosteroids.

The nurse is caring for a client receiving intravenous (IV) alteplase for a cerebrovascular accident (CVA). The nurse understands that this medication has reached its therapeutic effect when the client is assessed to have A. increase in the Glasgow Coma Scale. B. unintelligible speech. C. bleeding at their gum line. D. increase in pulse and decrease in blood pressure.

Explanation Choice A is correct. An increase in the Glasgow Coma Scale (GCS) is a favorable finding when tPA is administered (intravenous alteplase) for an ischemic stroke. The highest score on a GCS is 15. Choices B, C, and D are incorrect. Intravenous alteplase is a potent thrombolytic indicated for ischemic strokes and adversely may cause angioedema or severe bleeding. While slight bleeding at the gum line is not necessarily a reason to stop the infusion, it is not a therapeutic finding. Unintelligible speech would also be a consistent manifestation of a CVA that is not therapeutic when tPA is administered. An increase in pulse and decrease in blood pressure would suggest internal bleeding, which requires immediate follow-up by the nurse. Additional Info Alteplase is a thrombolytic indicated in the treatment of an ischemic stroke. To administer alteplase, the nurse must ensure that all invasive procedures are completed before the infusion to avoid bleeding. Two peripheral vascular access devices are needed, along with close monitoring of the client's vital signs and neurological status. Last Updated - 22, Jul 2022

The nurse is observing a client with epilepsy have a sudden loss of muscle tone that lasts for a few seconds. The nurse is correct in identifying this as which of the following? A. Atonic seizure B. Tonic-clonic seizure C. Absence seizure D. Complex partial seizure

Explanation Choice A is correct. Atonic seizures are drop attacks or drop seizures that cause a sudden loss of muscle tone and result in the client collapsing. This is quite serious as this may cause a client to sustain an injury. Choices B, C, and D are incorrect. Tonic-clonic seizures are characterized by stiffening the muscles (tonic), then the client has muscle jerking (clonic). Absence seizures feature a brief staring gaze with an impaired level of consciousness. These are common in children and may occur multiple times throughout the day. Complex partial seizures cause an impairment in consciousness, so the client may exhibit automatisms such as lip-smacking or repeating certain words/phrases. Additional Info Seizures can be caused by various factors, such as severe electrolyte disturbances (hypocalcemia, hyponatremia, hypomagnesemia). Physical insults to the brain may also induce a seizure. Epilepsy itself is idiopathic, with the usual onset in childhood or adolescence. Nursing care aims to educate the client on taking prescribed anti-epileptics and safety measures such as wearing a helmet (if applicable), special pillows, and regular follow-up care.

When an elderly home health client suddenly develops delirium, what is the first thing the home health nurse should assess for? A. Drug intoxication B. Increased hearing loss C. Cancer metastases D. Congestive heart failure

Explanation Choice A is correct. Drug intoxication, from prescription or OTC medications, is more common in the elderly, due to slower metabolism and absorption. Combinations of digoxin, diuretics, analgesics, and anticholinergics should be examined. Delirium is an acute and reversible syndrome. It is characterized by changes in memory, judgment, language, mathematical calculation, abstract reasoning, and problem-solving ability. The most common causes of delirium are infection, medications, and dehydration. Some symptoms of delirium include: Hallucinations Restlessness, agitation, or combative behavior Calling out, moaning, or making other sounds Being quiet and withdrawn — especially in older adults Slowed movement or lethargy Disturbed sleep habits Reversal of night-day sleep-wake cycle Choices B, C, and D are incorrect. Although the other options can lead to delirium, the onset is gradual, not sudden. NCSBN Client Need Topic: Psychosocial Integrity, Subtopic: Health Assessment and Promotion

Which of the following is the earliest indication of change in a child's neurological status? A. Level of consciousness B. Glasgow Coma Scale C. ICP measurement D. Pupil assessment

Explanation Choice A is correct. However, a simple observation, the level of consciousness, is widely recognized as the most sensitive indicator to a change, either improvement or deterioration, in a child's neurological status. Any changes, such as increased irritability or lethargy, should be reported to the team. Choice B is incorrect. The Glasgow Coma Scale evaluates eye-opening, motor response, and verbal response. It is an indicator of neurological status but is not the earliest indication of change. Choice C is incorrect. Direct ICP measurements are possible through a catheter inserted directly into the epidural space, but changes in ICP will manifest later than changes in a child's level of consciousness. Choice D is incorrect. A pupil assessment is an essential component of a neurological exam, but changes in the pupils would be a very late sign of an issue. The level of consciousness, on the other hand, will be a very early indication of change. NCSBN Client Need: Topic: Physiological Integrity, Subtopic: Physiological Adaptation, Neurologic

A client is brought to the emergency department due to vomiting, fever, and a severe headache. The physician suspects meningitis; then assesses the client for meningeal irritation and spinal nerve root inflammation. The nurse documents a positive Kernig's sign when: A. The client complains of pain when his knee is extended with his hip flexed. B. The client has a stiff neck when the neck is flexed towards the chest. C. The client's forearm spasms when a blood pressure cuff is inflated on the upper arm. D. The client feels pain in the calf when his foot is dorsiflexed.

Explanation Choice A is correct. Kernig's sign is positive if pain occurs upon extension of the knee. When meninges are inflamed (meningitis), movement of the spinal cord or nerves against the inflamed meninges results in pain. With the patient placed supine and hip flexed at 90 degrees, an extension of the knee stretches the hamstring and triggers pain by pulling tissues surrounding an inflamed spinal canal and meninges. Choice B is incorrect. Brudzinski's sign is also a sign of meningeal irritation/inflammation. Brudzinski's is positive when severe neck stiffness occurs after the neck is flexed towards the chest, causing the patient's hips and knees to flex. Choice C is incorrect. The appearance of a carpopedal spasm (flexion of the wrist, thumb, and metacarpophalangeal joints along with hyperextension of the interphalangeal joints) is referred to as Trousseau's sign. Such carpopedal spasms result from ischemia secondary to compression by the inflated sphygmomanometer cuff on a client's arm. Trousseau's sign signifies latent tetany; this is seen in hypocalcemia and hypomagnesemia. A positive Trousseau helps the clinician to detect the early presentation of hypocalcemia. Choice D is incorrect. When a client experiences pain in the calf upon sharp dorsiflexion of the foot with the knee extended, it refers to a Homan's sign. A positive Homan's indicates that the patient may have a deep vein thrombosis (DVT). However, Homan's by itself is considered an unreliable sign for diagnosing DVT and must be used in conjunction with diagnostic imaging and/or other physical assessment findings (Well's score). Learning objective: Meningitis refers to inflammation of the meninges, and certain signs such as Kerning's and Brudzinski's help detect meningeal inflammation on physical exam. NCSBN Client need: Topic: Physiological Integrity; Sub-Topic: Physiological adaptation/Infectious diseases

When a client with mid-stage Alzheimer's disease becomes agitated, which intervention should the nurse use? A. Placing an arm around the client's shoulders B. Turning on the television C. Place the client in a darkened room D. Encourage the client to join a group activity

Explanation Choice A is correct. Nursing interventions for Alzheimer's patients with "agitation" include providing a safe environment free of external stimulation and offering calming emotional support. Therapeutic touch is a non-pharmacological intervention that is an appropriate and effective treatment of agitation in patients with dementia. Placing an arm around the shoulders is comforting and provides reassurance to an agitated patient. Several studies have provided good evidence for the beneficial effects of regular therapeutic touch on reducing agitation in demented patients. Choices B, C, and D are incorrect. When a client with Alzheimer's disease becomes agitated or hostile, the nurse should respond in a calm and supportive way. Decreasing external stimuli will help lower the patient's agitation level. Turning on the television (Choice B) and leading the client to a group activity (Choice D) are inappropriate because they increase external stimulation and worsen agitation. Finally, the client should not be left alone in a darkened room (Choice C), as this may cause fear and result in increased agitation. Learning Objective Understand that therapeutic touch is an effective modality in reducing agitation in Alzheimer's clients. Additional Info Alzheimer's patients often exhibit behavioral symptoms. Such symptoms include agitation/restlessness, disruptive vocalization (screaming), pacing, sleeplessness, or aggression. Agitation in Alzheimer's patients can occur from various causes or the disease process itself. Identifying the cause of agitation (pain, drug interactions, infection) is crucial to managing the agitation effectively. Caregivers and health care providers use various interventions to treat these behavioral symptoms. However, these conventional interventions are fraught with safety problems and limited effectiveness. Examples include chemical and physical restraints, which can lead to an increased risk for falls. Therefore, therapeutic touch has been studied and established as an effective intervention in addressing agitation. Therapeutic touch is a very effective non-verbal communication technique that can offer immediate security and reassurance. The following guidelines are effective in managing agitation effectively:

A nursing assistant is feeding a patient with Parkinson's disease who is on aspiration precautions. Which action would require immediate intervention by the nurse? A. The nursing assistant reminds the client to keep his head back when he chews and swallows. B. The nursing assistant maintains the thickened liquid diet as ordered by the physician. C. The nursing assistant waits for the patient to finish swallowing before offering another bite. D. The nursing assistant does not offer fluids until the end of the meal.

Explanation Choice A is correct. Patients who are at risk for aspiration should be encouraged to swallow with their chin down. The nurse would need to intervene and give direct instructions about the proper way to chew and swallow to prevent aspiration. The risk for aspiration is applied when any patient has increased chances of secretions, solids, or fluids entering the tracheobronchial passages. Following physician orders for and ensuring the patient's food is at the ordered consistency is crucial. Nursing assistants and other ancillary personnel who may feed the client should be instructed on the proper way to feed, which includes allowing the patient to take his/her time and to make sure all food is swallowed before offering another bite. Fluids should be held until the end of the meal, when possible. Choices B, C, and D are incorrect. None of these require immediate nursing intervention. NCSBN Client Need Topic: Physiological Integrity, Subtopic: Reduction of Risk Potential

The nurse assesses a client with damage to cranial nerve III. Which finding would be expected? A. Ptosis B. Anosmia C. Uvula deviation D. Asymmetric facial movement

Explanation Choice A is correct. Ptosis, or eye drooping, occurs with cranial nerve III (oculomotor) lesions, myasthenia gravis, and Horner syndrome. Dysfunction of cranial nerve III is also associated with dilated pupil, absent light reflex, and impaired extraocular muscle movement. Choices B, C, and D are incorrect. Anosmia, or a decrease or loss in smell, occurs with dysfunction of cranial nerve I (olfactory), frontal lobe lesion, tobacco or cocaine use, allergic rhinitis, and upper respiratory infections. Uvula deviation to one side occurs with dysfunction of cranial nerve X (vagus), brainstem tumors, and neck injury. Unilateral facial movement occurs with dysfunction of cranial nerve VII (facial), central nervous system lesions such as stroke, and peripheral nervous system lesions such as Bell's palsy. Additional Info The twelve cranial nerves include - CN I: Olfactory CN II: Optic CN III: Oculomotor CN IV: Trochlear CN V: Trigeminal CN VI: Abducens CN VII: Facial CN VIII: Vestibulocochlear CN IX: Glossopharyngeal CN X: Vagus CN XI: Accessory CN XII: Hypoglossal Last Updated - 16, Feb 2022

A 45-year-old man on the neurology floor can understand instructions but is unable to express himself through talking. Which lobe of the brain controls the expression of speech? A. Frontal lobe B. Parietal lobe C. Temporal lobe D. Occipital lobe

Explanation Choice A is correct. The expression of speech is controlled by Broca's area in the frontal lobe. Broca's area is in the left hemisphere near the motor strip. Choice B is incorrect. The parietal lobe controls the sensory interpretation area for the opposite side of the body. It also controls the ability to interpret spoken words, the ability to read and write, and the ability to see body images. Choice C is incorrect. The temporal lobe controls primary auditory functions. It also controls Wernicke's area, which is the ability to organize language and understand what someone is saying. It also controls the limbic system and the hippocampus, which control emotions and memory, respectively. Choice D is incorrect. The occipital lobe controls visual processing and visual association. NCSBN Client Need Topic: Physiological Adaptation, Sub-topic: Pathophysiology, Nervous System

You are caring for a 12-year-old patient with a history of seizures. During her stay, you notice that she begins staring blankly. During this period, you are unable to get her attention, and she does not speak. You suspect that this is a: A. Petit mal seizure B. Simple partial seizure C. Grand mal seizure D. Myoclonic seizure Submit Answer

Explanation Choice A is correct. The petit mal (or absence) seizure is characterized by blank staring and an impaired level of consciousness. This type of seizure usually begins between the ages of 3 and 15 years. Choice B is incorrect. In the simple partial (or Jacksonian) seizure, the patient will be in an awake state but will exhibit abnormal motor or autonomic behaviors that can affect any part of the body. Choice C is incorrect. The grand mal (or tonic-clonic) seizure is the type of seizure in which there is a rapid extension of the arms and legs with sudden jerking and eventual loss of consciousness of the patient. It is often accompanied by incontinence and post-ictal confusion. Choice D is incorrect. During the myoclonic seizure, the patient may be awake or with short periods of loss of consciousness. During this seizure, the patient will have abnormal motor behavior in one or more muscle groups that lasts a few seconds to a few minutes. NCSBN Client Need Topic: Physiological Adaptation, Sub-Topic: Pathophysiology, Neurologic

The nurse observes a novice nurse caring for a client experiencing status epilepticus. It will require immediate intervention if the novice nurse does which of the following? A. Prepares to administer intravenous valproate. B. Places the client in a lateral position. C. Activates the rapid response team (RRT). D. Loosens any restrictive clothing.

Explanation Choice A is correct. This action requires follow-up because, during an acute seizure or status epilepticus, intravenous benzodiazepines such as lorazepam, diazepam, or midazolam should be promptly administered to this client. These medications help terminate the seizure. IV antiepileptics such as valproate, topiramate, and phenytoin should be used secondary only after the acute seizure has terminated. Thus, a medication used to prevent seizure reoccurrence rather than medication used to terminate a seizure is not the priority. Choices B, C, and D are incorrect. Status epilepticus is a medical emergency, and notifying the RRT is appropriate considering the severity of this situation. Loosening restrictive clothing and placing the client on their side will help decrease the risk of aspiration and injury. Additional Info Status epilepticus is a medical emergency. This is when a seizure has lasted five or more minutes. Additionally, status epilepticus is defined as two or more continuous seizures without complete recovery. During an acute seizure, the nurse should place the client on their side, loosen restrictive clothing, and anticipate a prescription for a parenteral benzodiazepine such as diazepam.

The nurse is supervising a student nurse caring for a newborn. Which of the following reflexes, if elicited by the student nurse, would be the plantar reflex? Correct

Explanation Choice A is correct. This image shows the plantar reflex, which is also known as the Babinski reflex. The plantar reflex occurs after the sole has been stroked upwards. It causes the big toe to move upward and then the other toes fan out. Choice B is incorrect. This image shows the rooting reflex. This reflex is seen in normal newborn babies. It occurs when they turn their face toward stimulation (such as stroking their cheek) and make sucking motions with the mouth. These sucking motions are what is referred to as root. This reflex helps to ensure successful feeding. Choice C is incorrect. This image shows the stepping reflex. The stepping reflex in newborns is seen when a baby is held upright, and the baby's feet are touching the ground. Their feet will make stepping motions as if they are walking. This reflex is prevalent from birth until 2 to 3 months. Choice D is incorrect. This image shows the tonic neck reflex. This reflex is elicited when the head is turned to one side. The baby will extend their arm and leg on the side that the head is turned towards while the opposite arm and leg bend. This reflex dissipates by six months of age. Additional Info To assess for the Babinski reflex, stroke the lateral sole of the foot from the heel to across the base of the toes. Child < 1 year of age: Babinski present --> great toe bends upward, and other toes fan out. Child > 1 year of age and adults: Babinski absent --> Plantar flexion of the toes ( normal)

A nurse is caring for a post-cardiovascular accident (CVA) client who continues to have slight cognitive difficulties, specifically exhibited via intermittent difficulty understanding verbal communication. Which action by the nurse is most appropriate when talking to this client? A. Speak slowly with simple directions and gestures B. Speaking to the client in a raised tone of voice C. Encouraging the client to respond to every statement by the nurse D. Consistently shifting topics of conversation

Explanation Choice A is correct. When providing care for a client experiencing post-CVA cognitive difficulties, nurses should adapt communication to maximize understanding. Ideally, communication should be spoken slightly slower than normal and include simple directions and gestures to facilitate comprehension by the client. By utilizing effective communication techniques, the nurse not only builds rapport with the client but assists in building the client's confidence. Choice B is incorrect. The nurse should talk to the client in a soft, audible voice. The client's issue is not related to an inability to hear the nurse's words; the issue is the client's ability to understand what is being said. Choice C is incorrect. Placing pressure on the client to respond to each statement made by the nurse will place undue pressure on the client, leading to frustration and potential setbacks in the client's treatment. Choice D is incorrect. The nurse should avoid frequent and/or abrupt shifts in conversation topics. Conversations should flow in a natural manner where topics gradually shift topics. When necessary, the nurse should introduce a new topic to the client to decrease confusion on the client's part. Learning Objective Recognize that some post-CVA clients may have slight cognitive difficulties requiring nurses to speak slowly and provide simple directions with gestures to facilitate understanding.

Select an appropriate nursing diagnosis for your client who is affected with hyperalgesia. A. At risk for inadvertent narcotic overdoses related to hyperalgesia. B. At risk for abnormal and irreversible pain related to hyperalgesia. C. At risk for somatic pain related to hyperalgesia. D. At risk for visceral pain related to hyperalgesia.

Explanation Choice B is correct. "At risk for abnormal and irreversible pain related to hyperalgesia" is an appropriate nursing diagnosis for a client who is affected with hyperalgesia. Hyperalgesia, which is synonymous with hyperpathia, is abnormal pain processing that can lead to the appearance of neuropathic pain that is irreversible if left untreated. Choice A is incorrect. "At risk for inadvertent narcotic overdoses related to hyperalgesia" is not an appropriate nursing diagnosis for a client who is affected with hyperalgesia. Hyperalgesia is abnormal pain processing that is not associated with inadvertent narcotic overdosages. Choice C is incorrect. "At risk for somatic pain related to hyperalgesia" is not an appropriate nursing diagnosis for a client who is affected with hyperalgesia. Hyperalgesia can lead to neuropathic pain, but not somatic nociceptive pain. Choice D is incorrect. "At risk for visceral pain related to hyperalgesia" is not an appropriate nursing diagnosis for a client who is affected with hyperalgesia. Hyperalgesia can lead to neuropathic pain, but not visceral nociceptive pain. Last Updated - 20, Jan 2022

A client is admitted with a possible diagnosis of Guillain-Barré syndrome. Which important question should the nurse include when taking this client's history? A. "Do you experience frequent bruising?" B. "Did you have a recent respiratory or gastrointestinal infection?" C. "Have you been overseas during the past four months?" D. "Has anybody in your family had Guillain-Barré syndrome?"

Explanation Choice B is correct. Approximately 60-70% of clients diagnosed with Guillain-Barré syndrome experienced the onset of symptoms approximately one to four weeks following a respiratory or gastrointestinal tract infection. Although the cause of Guillain-Barré syndrome is not fully understood, it is thought to be autoimmune. Choice A is incorrect. Guillain-Barré syndrome does not affect the body's ability to bruise. Choice C is incorrect. Guillain-Barré syndrome is not related to foreign travel. Choice D is incorrect. There is no evidence that Guillain-Barré syndrome is a hereditary disorder. Learning Objective When speaking with a Guillain-Barré syndrome client, recognize the importance of inquiring about recent respiratory and/or gastrointestinal infections. Additional Info Guillain-Barré syndrome typically begins with an ascending, relatively symmetric flaccid weakness. Most clients improve considerably over a period of months, but about 30% of adults and even more children have some residual weakness at the three-year mark, and 2-5% develop chronic inflammatory demyelinating polyneuropathy. Intensive supportive care is key to recovery.

The nurse is assisting the physician with a lumbar puncture to assess for meningitis. What should be the first nursing action of the nurse? A. Lay the client on his side. B. Ask the client to void. C. Obtain an advanced directive from the client. D. Withhold food and drinks from the client prior to the procedure.

Explanation Choice B is correct. As an additional comfort measure, the client is asked by the nurse to empty his bladder before the procedure begins. This should be the first action of the nurse. Choice A is incorrect. The client should be laying on his side in a "C" position. His back should be as close to the edge of the bed as possible. However, this is not the first intervention. Choice C is incorrect. It is best for the client to have an advanced directive should he be unable to decide for himself; however, this is not a necessity for this procedure. Choice D is incorrect. The procedure does not require the client to be NPO (nothing by mouth). Last Updated - 18, Oct 2021

The nurse suspects a patient on the neurological floor is experiencing autonomic dysreflexia. What action would be the nurse's priority? A. Administer sublingual nitroglycerin. B. Elevate the head of the bed. C. Obtain a residual volume reading with a bladder scan. D. Perform a digital examination to assess for the presence of stool.

Explanation Choice B is correct. Autonomic dysreflexia is a severe, life-threatening condition that can occur secondary to a spinal cord injury. In response to a noxious stimuli such as full bladder, line insertion, or fecal impaction, the body mounts an exaggerated sympathetic response that causes bradycardia, hypertension, facial flushing, and headache. If left untreated, autonomic dysreflexia can cause cerebral hemorrhage, pulmonary edema, and seizures. Treatment is focused on removing the underlying noxious stimuli. The nurse should first sit this patient up to attempt to reduce blood pressure quickly, begin frequent vital checks, loosen restrictive clothing, then assess for full bladder or fecal impaction. Choice A is incorrect. This would not be the initial priority for a patient experiencing autonomic dysreflexia. If systolic blood pressure is over 150 mmHg, it may be appropriate to administer a rapid-onset, short duration antihypertensive medication, but further assessment should be done first. Choice C is incorrect. Scanning the bladder would not be a priority for a patient experiencing autonomic dysreflexia. If an indwelling catheter is not in place, the nurse should insert one, and if already in place, the nurse should assess for kinks in tubing, attempt to irrigate the bladder, and if still not draining, then replace the catheter. Choice D is incorrect. This would not be the initial priority for a patient experiencing autonomic dysreflexia. The most common cause of autonomic dysreflexia is related to bladder distention, so the highest priority would be to assess the urinary system. If symptoms do not improve, this action would be an appropriate next step and if stool is present, the nurse should gently remove and reassess. Last Updated - 08, Feb 2022

The nurse is caring for a patient with a T5 spinal cord injury. Which assessment information would indicate to the nurse that the patient is experiencing autonomic dysreflexia? A. Distended bladder B. Headache C. Hypotension D. Generalized pallor

Explanation Choice B is correct. Autonomic dysreflexia is a severe, uncompensated cardiovascular reaction that occurs in response to visceral stimulation after spinal shock has resolved. Patients with spinal cord injuries at T6 or above are at risk of developing autonomic dysreflexia. A sudden, throbbing headache is one of the most important warning symptoms that a patient is experiencing this life-threatening condition. Choice A is incorrect. Bladder or rectum distention is the most common cause of autonomic dysreflexia, not a symptom experienced as a result. In addition, bladder distention could indicate a number of other issues and would not be specific to autonomic dysreflexia. Choice C is incorrect. Autonomic dysreflexia is associated with an elevation in blood pressure and/or reduction in heart rate, not reduced blood pressure. Choice D is incorrect. Autonomic dysreflexia causes flushed face and upper chest above the level of the lesion and pale extremities below the level of the lesion, not generalized paleness over all areas of the body. Last Updated - 16, Feb 2022

What is the term that is used to describe a human's innate biological clock relating to daytime and nighttime wakefulness and activity? A. REM sleep B. Circadian rhythm C. Diurnal activity D. Nocturnal activity

Explanation Choice B is correct. Circadian rhythm is defined as our 24-hour biological clock that is primarily one that functions best with daytime wakefulness/activity and nighttime sleep. When clients are in synchrony with their biological clock, humans function optimally because many of our essential rational physiological and mental functions like blood pressure, body temperature, and levels of alertness/performance are at their optimal levels. Choice A is incorrect. REM sleep is a phase of the sleep cycle and not the term that is used to describe a human's innate biological clock relating to daytime and nighttime wakefulness/activity. Choice C is incorrect. Diurnal activity is daytime activity and not the term that is used to describe a human's innate biological clock relating to daytime and nighttime wakefulness/activity. Choice D is incorrect. Nocturnal activity is nighttime activity and not the term that is used to describe a human's innate biological clock relating to daytime and nighttime wakefulness/activity. Last Updated - 29, Jul 2021

A client admitted to the medical ward for convulsions is receiving intravenous magnesium sulfate. Which of the following signs indicate an expected side effect of the drug? A. Less frequency of urination B. Frequent sleepiness C. Absence of a knee jerk reflex D. Decreased respirations

Explanation Choice B is correct. Clients taking magnesium sulfate are expected to become sleepy during the daytime as well as experience hot flashes and lethargy. Choice A is incorrect. Magnesium prevents or controls convulsions by blocking neuromuscular transmission and decreasing the amount of acetylcholine liberated at the endplate by the motor nerve impulse. Magnesium sulfate does not affect urine production. Choice C is incorrect. The absence of deep tendon reflexes indicates elevated magnesium levels. As plasma magnesium rises above 4 mEq/liter, the deep tendon reflexes are decreased. Choice D is incorrect. This indicates magnesium toxicity. As the plasma level approaches 10 mEq/liter, respiratory paralysis may occur. A decrease in respiratory rate initially manifests this. Last Updated - 21, Jan 2022

The nurse is assessing a patient's lower extremities for clonus. What would the nurse recognize as a positive result? A. Unilateral redness and swelling that is warm to the touch. B. Rapid, rhythmic muscle contractions. C. Popping or clicking of the knee joint with movement. D. Audible cracking and palpable grating with movement of the joints.

Explanation Choice B is correct. Clonus is an abnormal response to deep tendon reflex stimulation that is characterized by rapid, rhythmic muscle contractions. Choice A is incorrect. Unilateral inflammation would be a possible sign of cellulitis, a skin condition caused by infection of the soft tissue. Choice C is incorrect. Popping or clicking of the knee would indicate damage to fibrocartilage in the knee or meniscal injury. Choice D is incorrect. Audible cracking and palpable grating with the movement of joints describe crepitation (crepitus). This finding would indicate fracture, dislocation, or osteoarthritis. NCSBN Client Need Topic: Musculoskeletal, Subtopic: System-specific assessments, pathophysiology Last Updated - 20, Jan 2022

The nurse is working with a child who has a learning disability. The child is ten years old and has trouble reading and interpreting words, letters, and symbols. What is the most likely diagnosis? A. Phonologic processing deficit B. Dyslexia C. Tourette's syndrome D. Apraxia

Explanation Choice B is correct. Dyslexia is defined as a disorder that involves trouble reading and interpreting words, letters, and symbols. It does not affect general intelligence, but children may need special assistance at school when learning to read. They may not understand at their appropriate grade level, depending on the severity of the disorder. Choice A is incorrect. Phonologic processing deficit is not described as trouble reading and interpreting words, letters, or symbols. Instead, it is a specific deficit where the child has difficulty discriminating and processing different speech sounds. Choice C is incorrect. Tourette's syndrome is a neurological disorder characterized by involuntary tics and vocalizations. It is often best known for the compulsive utterance of obscenities that sometimes occurs. Tourette's syndrome is not characterized by trouble reading and interpreting words, letters, or symbols. Choice D is incorrect. Apraxia is defined as the inability to perform particular purposive actions, as a result of brain damage. It is not a learning disability and is not characterized by trouble reading and interpreting words, letters, or symbols. NCSBN Client Need: Topic: Psychosocial Integrity; Subtopic: Pediatrics - Neurology Last Updated - 09, Feb 2022

The nurse is assessing a patient's neurological status and notes 4+ deep tendon reflexes (DTR). Which of the following conditions would not be a possible cause of hyperactive DTRs? A. Hypocalcemia B. Muscular dystrophy C. Upper motor neuron lesion D. Hyperthyroidism

Explanation Choice B is correct. Hyperactive deep tendon reflexes (DTRs) would not be expected in a patient with muscular dystrophy. Muscular dystrophy DTRs are typically decreased or absent. Choice A is incorrect. Hypocalcemia (low calcium level) is a potential cause of hyperactive DTRs. Choice C is incorrect. The presence of an upper motor neuron (UMN) lesion is a potential cause of hyperactive DTRs. Choice D is incorrect. Hyperthyroidism is a potential cause of hyperactive DTRs. NCSBN Client Need Topic: Skills/procedures, Subtopic: potential for complications from health alterations, system-specific assessments Last Updated - 13, Feb 2022

A nurse is caring for a client with multiple sclerosis (MS) undergoing plasmapheresis. The nurse understands that plasmapheresis controls symptoms of MS by removing which of the following from the blood? A. Catecholamines B. Antibodies C. Plasma proteins D. Lymphocytes

Explanation Choice B is correct. In clients with multiple sclerosis, an autoimmune reaction occurs. This autoimmune reaction causes immune cells and antibodies to attack and destroy the myelin sheath (a coating made of fat and proteins that protects nerves and helps transmit electrical signals) and the underlying nerve fibers in the brain, optic nerves, and spinal cord. During plasmapheresis, these antibodies are removed from the client's plasma, removing the cause of myelin sheath demyelination. Choice A is incorrect. Catecholamines are the "fight or flight" group of hormones released by your adrenal glands in response to physical or emotional stress. These hormones include epinephrine (adrenaline), norepinephrine, and dopamine. Any unused hormone(s) in the body is excreted through the urinary system. Plasmapheresis does not remove catecholamines. Choice C is incorrect. Plasmapheresis is a process that filters the blood and removes harmful antibodies. It does not remove plasma proteins. Choice D is incorrect. Lymphocytes are not removed during plasmapheresis. Learning Objective Understand that the purpose of plasmapheresis use in multiple sclerosis (MS) clients is to remove antibodies from the client's blood. Additional Info The cause of multiple sclerosis (MS) is unknown but likely involves an attack by the immune system against the body's own tissues (i.e., an autoimmune reaction). In most multiple sclerosis clients, periods of relatively good health alternate with episodes of worsening symptoms. MS is a progressive disease, as symptoms gradually worsen over time. Clients may experience visual problems, abnormal sensations, and weak or clumsy movements. Diagnosis is based on clinical symptoms, physical examination, and magnetic resonance imaging. Treatment includes corticosteroids, immunomodulators to prevent exacerbations and delay eventual disability, antidepressants, and/or supportive care. Life span is typically unaffected unless the disorder is very severe.

The nurse gives discharge instructions to a client who sustained a brain injury from a motor vehicle accident. His wife is concerned regarding her husband having seizures at home. Which statement from the wife indicates that she understood the nurse's teaching? A. "I will make sure that my husband does not wet himself." B. "I will clear all furniture that might injure him when he has a seizure." C. "I will call 911 once he has a seizure lasting about 3 minutes." D. "I will ensure he sleeps well after a seizure."

Explanation Choice B is correct. One of the major goals during a seizure is injury prevention. Caregivers should be taught about injury prevention precautions. The wife should ensure that the furniture is moved out of the way when her husband seizes, improving his safety. Choice A is incorrect. There is a chance for the client to urinate while having a seizure. However, the wife does not have any control over his urinary incontinence unless the client wears incontinence aids. The priority should be placed on injury prevention, not urinary incontinence. Choice C is incorrect. Self-limiting seizures are not life-threatening. The wife need not call 911 unless the seizure lasts longer than 5 minutes. Status Epilepticus is defined as a single seizure lasting more than five minutes or two or more seizures occurring within a five-minute period without the person returning to normal between them. The wife should be educated that status epilepticus is a medical emergency and she should seek help if such an event occurs. Choice D is incorrect. It is essential that the client rests after his seizure. However, the caregiver's priority concern among the given options is injury prevention.

The nurse is caring for a client who sustained an ischemic cerebrovascular accident (CVA) three hours ago. The client's most recent blood pressure was 168/101 mm Hg. The nurse should take which action? A. Place the client supine B. Continue to monitor C. Obtain orthostatic blood pressure D. Request a prescription for an antihypertensive

Explanation Choice B is correct. Permissive hypertension during an ischemic stroke allows the blood pressure to go up to 185/110 mm Hg. This enables perfusion around the stroke to distal tissue. A blood pressure of 150/100 mm Hg is needed to maintain cerebral perfusion after an acute ischemic stroke; pressures above this reading may lead to an extension of the stroke. Choice A, C, and D are incorrect. Placing the client supine during a stroke is contraindicated because of its increase in the intracranial pressure. Orthostatic blood pressure is not indicated and is usually performed if hypovolemia could cause hypotension, not hypertension. An antihypertensive is not necessary based on this blood pressure.

The nurse is preparing to teach a client who was recently diagnosed with Meniere's disease. To help the client reduce the incidence of attacks, the nurse should recommend that the client do which of the following? A. Irrigate their ear with sterile water. B. Reduce dietary sodium intake. C. Not use earbuds or headphones. D. Speak with limited inflections.

Explanation Choice B is correct. Reducing dietary sodium intake is key to reducing attacks associated with Meniere's disease. By reducing sodium, the client will reduce endolymphatic fluid, reducing the incidence of attacks. Choices A, C, and D are incorrect. Irrigating the ear with sterile water is not recommended to prevent attacks. The inner ear is implicated in Meniere's disease, specifically, the accumulation of endolymphatic fluid. Earbuds and headphones should be used for a client with Meniere's, as hearing loss is one of the features of this disease. These devices may assist a client with amplifying their hearing. Speaking with limited speech inflections would have no bearing on this disease. Additional Info Meniere's disease is characterized by excessive endolymphatic fluid. This causes three main features of vertigo, tinnitus, and sensorineural hearing loss. Nursing care aims to provide education that should focus on diet medication (low salt, limiting caffeine and alcohol) and adherence to pharmacotherapy, including antiemetics, diuretics, antihistamines, and glucocorticoids. Last Updated - 03, Dec 2022

The nurse in the neurology ward is taking care of a client with paraplegia due to a spinal cord injury. The nurse is planning for the client's rehabilitation. Which would be the most effective plan for the patient? A. The patient and the family will have to arrange for the rehabilitation. B. The plan should be implemented early on in the treatment of the patient. C. The patient should plan for minimal and short-term rehabilitation as he will be able to return to his former activities. D. Long-term care should be arranged because the client is no longer capable of self-care

Explanation Choice B is correct. Rehabilitation should start early in the treatment. This provides the patient with an optimistic atmosphere and makes the transition to discharge a lot easier. Choice A is incorrect. The client and the family are not familiar with the options available in the health care system; the nurse should provide information and support to both the client and his family. Choice C is incorrect. The client is being prepared to adapt to a new lifestyle and will have to adjust to his paralysis. Choice D is incorrect. Self-reliance and independence are the goals of rehabilitation. Last Updated - 12, Jan 2022

The nurse is caring for a patient with Meniere's disease. Which of the following nursing interventions is of the highest priority when caring for this patient? A. Discussing treatment options B. Initiating fall risk measures C. Keeping the patient calm during an episode D. Providing teaching on potential causes

Explanation Choice B is correct. Since Meniere's disease causes vertigo or the feeling that one is spinning, the patient is at an increased risk for falls. To keep this patient safe, the nurse must initiate fall risk measures. Choice A is incorrect. Discussing treatment options, while necessary, is not the highest priority when caring for patients with Meniere's disease. Promoting patient safety by preventing falls is a priority. Choice C is incorrect. While an episode of vertigo can be frightening in a patient, keeping the patient calm is not the highest priority when caring for patients with Meniere's disease. Promoting patient safety by preventing falls is a priority. Choice D is incorrect. Providing teaching on causes of episodes is always an essential part of patient care; however, keeping the patient with Meniere's disease safe should be the priority. NCSBN client need Topic: Phys

After a patient experiences a motor vehicle accident (MVA) and suffers a complete spinal cord injury to L3, the nurses would assess for loss of motor function in the: A. Abdomen B. Arms C. Legs D. Chest

Explanation Choice C is correct. The level of injury in the spinal cord correlates with innervation on the skin according to the level of the dermatome. Choice A is incorrect. Innervation of the abdomen corresponds to T9 to T12 injury. Choice B is incorrect. Innervation of the arm correlates with C5 to T1. Choice D is incorrect. Injury to T1-T8 correlates with chest innervation. NCSBN Client Need Topic: Physiological Integrity, Subtopic: Reduction of Risk Potential; Spinal Cord Injuries Last Updated - 08, Feb 2022

What does the area labeled 13 in the picture below represent? A. The anterior horn B. The dorsal root ganglion C. The anterior root D. The posterior root

Explanation Choice B is correct. The dorsal root ganglion is labeled 13 in the picture. A ganglion is a collection of cell bodies of the neurons outside the central nervous system. This cluster of cell bodies in the dorsal root ganglion gives rise to all the fibers present in the spinal nerve's dorsal root, which carries afferent ( sensory) information to the spinal cord. The dorsal/ posterior root nerve fibers sense painful and noxious stimuli that can be chemical or thermal. Dorsal ( posterior) roots and ventral ( anterior) roots are the components of spinal nerves. There are 31 pairs of spinal nerves, one on each side of the vertebral column. Spinal nerves are considered mixed nerves because they carry afferent ( sensory), efferent ( motor), and autonomic signals between the spinal cord and the body. Dorsal roots carry afferent ( sensory) impulses, whereas ventral roots carry efferent (motor) signals. A reflex arc refers to a pathway followed by these nerve impulses in the case of a reflex action. A reflex action is an involuntary protective response to a painful stimulus leading to the withdrawal of the body away from the painful impulse. The dorsal root ganglion forms the sensory component of this reflex arc, whereas the ventral roots form the motor component. For example, when the client touches a very hot object, he will immediately withdraw the hand from the object ( reflex action). In this action, the pain perception is mediated by the dorsal root ganglion. If the dorsal root ganglion is diseased or absent, the reflex action will not occur properly, and injury will worsen to the client's hand. Alternatively, an inflammation of the dorsal root ganglion ( ganglionitis) may cause severe and chronic pain. Dorsal root ganglionitis may be seen in trauma, sciatica, disc herniation, neuropathies, and viral infections such as Herpes Simplex Virus-1. Herpes zoster (shingles) results from reactivation of latent varicella-zoster virus infection within the dorsal root ganglia. Choices A and C are incorrect. The anterior horn is labeled 1, whereas the anterior root is labeled 11 in the picture. The ventral ( anterior) roots mediate the motor component ( efferent pathway) of the reflex arc. Choice D is incorrect. The posterior or dorsal root is labeled as 12 in the picture, and cell bodies of the dorsal root nerves are located in the dorsal root ganglion (marked 13). These dorsal anatomical structures shown in the image play a role in pain and pain perception.

The leading and single MOST important indicator of the intensity and presence of pain is: A. A quantitative assessment/measurement of pain with a pain intensity scale. B. The client's reports of pain to the nurse and other healthcare providers. C. A qualitative assessment/measurement of pain with a pain intensity scale. D. The nurse's observation and assessment of pain behaviors such as guarding and moaning.

Explanation Choice B is correct. The leading and single MOST crucial indicator of the intensity and presence of pain is the client's reports of pain to the nurse and other healthcare providers. Too often, pain is undertreated because we fail to listen to the client and their self-reports of pain; instead, we regularly assess and evaluate client vital signs and pain behaviors, which are less accurate and reliable than the client's reports. Choice A is incorrect. Although a quantitative assessment and measurement of pain with a pain intensity scale can and should be used, this is only one way to obtain the subjective client's reports of pain. Choice C is incorrect. Although a qualitative assessment can and should be used, this is only one way to obtain the subjective client's reports of pain, including its description and characteristics. Choice D is incorrect. Although the nurse will observe and assess pain behaviors such as guarding and moaning, this is not the leading and single most important indicator of the intensity and presence of pain. Last Updated - 26, Jan 2022

The nurse should assess an Alzheimer's patient who has been started on rivastigmine for which of the following side effects? A. Liver toxicity B. Weight loss C. Renal failure D. Extrapyramidal side effects

Explanation Choice B is correct. The most common side effects of rivastigmine are flu-like symptoms, dizziness, and weight loss. The FDA has approved limited drugs for Alzheimer's Disease. The most effective medications act by intensifying the effect of acetylcholine at the cholinergic receptor. Acetylcholine is naturally degraded in the synapse by the enzyme acetylcholinesterase. When acetylcholinesterase is inhibited, acetylcholine levels increase and significantly affect the receptors. Choice A is incorrect. Liver toxicity is not an anticipated side effect of rivastigmine. Choice C is incorrect. Renal failure is not an anticipated side effect of rivastigmine. Choice D is incorrect. Extrapyramidal symptoms are not an anticipated side effect of rivastigmine. NCSBN Client Need Topic: Physiological Integrity, Subtopic: Pharmacologic Intervention, Acetylcholinesterase inhibitors Used for Alzheimer's Disease Last Updated - 02, Jan 2022

A client is having a seizure when the nurse enters the room. What should be the most appropriate action of the nurse? A. Note the first area that starts to seize. B. Take note of the time the seizure began and how long it lasts. C. Step out of the room to quickly bring pads for the side rails. D. Provide privacy to the client during the seizure

Explanation Choice B is correct. The nurse should time the seizure to obtain additional information. The nurse's intervention requires staying in the room, assessing, and protecting the client. If the client is standing or sitting while having a seizure, the client should be gently laid down and turned to the side so that the tongue does not block the airway. The patient must not be restrained. If readily available, pillow and bed-pads should be used to protect the client's head and extremities, respectively. Most seizures last from 30 seconds to two minutes. The nurse should time the seizure and make sure it resolves because if the seizure lasts for more than 5 minutes or seizures occur repetitively without the person regaining consciousness between the seizures, then it is defined as "status epilepticus." Status epilepticus is a medical emergency. If the seizure lasts more than 5 minutes or another seizure occurs, the nurse must call for help and activate the rapid response team. The physician should be notified regarding any seizure event so the etiology can be investigated. Choice A is incorrect. Noting the first area that starts to seize is useful in determining the site of the seizure's origin in the brain. However, since the client has already been seizing when the nurse entered the room, the nurse will not be able to determine this. Choice C is incorrect. The nurse should stay in the room, ensure the client is protected, and time the seizure. Leaving the room to get padding for the side rails is inappropriate. If there is information to suggest that the client is at high risk for seizures, the side rail padding (seizure precautions) should be done at the time of admission. Choice D is incorrect. The client's privacy should always be ensured; however, ensuring the safety of the client and timing the seizure should take priority over privacy. Last Updated - 26, Jan 2022

Select the type of pain below that is accurately paired with one of its subjective sensory descriptors and one of its subjective affective, emotional descriptors. A. Ache: Radiating and throbbing B. Pain: Drilling and exhausting C. Hurt: Radiating and miserable D. Ache: Tender and throbbing

Explanation Choice B is correct. The type of pain that is accurately paired with one of its subjective sensory descriptors and its personal affective, emotional descriptors is "pain: drilling and exhausting." Other individual sensory descriptors of pain include "sharp, crushing, and wrenching"; other subjective affective, emotional descriptors of pain include "agonizing, unbearable, and torture." Ache is the least intense, hurt is the next level of intensity, and pain is the most intense. Choice A is incorrect. Although ache can be described with the sensory descriptor of "radiating," the ache is not related to the affective, emotional pain descriptor of "throbbing." Some useful emotional pain descriptors of ache are "annoying and tiring." Choice C is incorrect. Hurt is not related to the sensory descriptor of "radiating" or the affective, emotional pain descriptor of "miserable." Instead, hurt can be reported with the sensory descriptor of "pricking" and the affective, emotional pain descriptor of "throbbing." Choice D is incorrect. Ache is not related to the affective, emotional pain descriptor of "miserable." Instead, ache can be associated with the sensory descriptor of "cramping" and the affective, emotional pain descriptor of "tender."

A 24-year old woman presents to the emergency department and appears as shown in the exhibit. What type of injury does this assessment finding suggest? See the exhibit. A. CSF leak B. Basilar skull fracture C. Brown-sequard syndrome D. Subarachnoid hemorrhage

Explanation Choice B is correct. This picture represents a clinical assessment symptom called Raccoon's eyes (retroorbital ecchymosis). Pooling of blood surrounding the eyes is most often associated with fractures of the anterior cranial fossa or basilar skull fracture. This assessment finding may be delayed by 1 to 3 days following the initial injury. If bilateral, this sign is highly suggestive of a basilar skull fracture. Other signs of basilar skull fractures include hemotympanum (pooling of blood behind the tympanic membrane) and Battle sign (retro auricular or mastoid ecchymosis). Choice A is incorrect. Although Cerebrospinal fluid (CSF) leak could be a later complication of this injury, the initial injury that this patient is suffering from is a basilar skull fracture. A CSF leak occurs in about 20% of patients following a basilar skull fracture. When meningeal structures are damaged by the fractured bones, CSF can leak through the subarachnoid space and manifests as "clear fluid" draining from the nostrils (CSF rhinorrhea) or ears (CSF otorrhea). To confirm that the fluid is indeed CSF, the physician may order testing the fluid for beta-transferrin. Choice C is incorrect. The Brown-Sequard syndrome (BSS) is a hemisection of the spinal cord and does not cause Raccoon's eyes. Symptoms of the BSS include weakness and loss of proprioception on one side of the body (ipsilateral side of injury) and loss of temperature sensation on the opposite side. Causes for the BSS include a spinal cord tumor, trauma, ischemia, or infectious or inflammatory diseases (tuberculosis, or multiple sclerosis). Choice D is incorrect. Signs of a subarachnoid hemorrhage (SAH) include severe headache (often stated by the patients as "the worst headache of their life"), photophobia, nausea, vomiting, and vision changes. Causes of SAH include aneurysmal rupture or trauma. NCSBN Client Need Topic: Physiological Integrity, Subtopic: Neurologic Last Updated - 09, Dec 2021

The nurse is conducting patient teaching to a client with a level T4 spinal cord injury to transfer from the bed to the wheelchair independently. The nurse should emphasize to the client to move: A. His upper and lower body should move together into the wheelchair. B. His upper body moves into the wheelchair first. C. His lower body into the wheelchair first, placing his feet on the pedals, and then his hands to the wheelchair arms. D. His buttocks to the wheelchair first and then place his feet to the floor.

Explanation Choice B is correct. When transferring a patient with paralysis of the lower extremities from a bed to a wheelchair, move the big part of the body (upper) to the chair first. This is the proper technique and the safest. The client should move his upper body to the wheelchair first, then his legs from the bed to the wheelchair. Choices A, C, and D are incorrect. Other methods are not as secure and can lead to injury. Last Updated - 20, Jul 2022

You are a home health nurse caring for an elderly client in her home. She has children and grandchildren. However, they live far from the couple and they typically visit only once or twice a year. The client is beginning to show some signs of Alzheimer's. The husband is 88-years-old and had a stroke that left him with right-sided weakness. What support should you give the husband in terms of caring for his wife? A. You should advise the couple to move closer to their children so that they can care for their father. B. You should teach the husband about the progression of Alzheimer's and the need to promote as much independence as possible. C. You should teach the husband about this progressive disease and the need to do all that he can for his wife to help prevent anxiety and depression. D. You should advise the couple to decrease their social activities in order to preserve the wife's dignity and self-esteem.

Explanation Choice B is correct. You should teach the husband about Alzheimer's and the need to promote as much independence as possible. Adults diagnosed with dementia are faced with a disease that is irreversible and progressive. The loss of judgment, reasoning, memory, and communication skills leads to an inability to discern risk and danger. Dementia can limit a person's ability to live independently, which can be very distressing for the individual and family members. Caregivers need to embrace a patient-centered approach that allows people with dementia to maintain as much autonomy and control as possible, while still preserving their safety. Choice A is incorrect. Moving closer to the children may not be appropriate advice, mainly if the children are unable or unwilling to care for their mother. Choice C is incorrect. Client's with Alzheimer's disease and other disabilities, including physical disabilities, should be coached and encouraged to be as independent as possible. Choice D is incorrect. The couple should be advised to continue their social activities.

A nurse is caring for a client following a stroke diagnosis which rendered the client with stage I dysphagia. Which foods should the nurse feed the client based on the client's dysphagia severity? A. Peeled, ripe peaches B. Peas, squash, and cooked carrots C. Puréed lean meat and egg yolks D. Pies, cakes, and ice cream

Explanation Choice C is correct. A client with stage I dysphagia has severe difficulty swallowing. These clients must be fed puréed foods. Stage I dysphagia clients are fed diets consisting of primarily puréed foods, including puréed fruits, vegetables, and meats. Additional foods include gravies, puddings, egg yolks, and baby foods. Choice A is incorrect. A client with stage I dysphagia has severe difficulty swallowing. Peaches require the client to control the food in their mouth and cannot be given to a client with stage I dysphagia as they do not possess the muscle tone necessary to control the food. Choice B is incorrect. A client with stage I dysphagia has severe difficulty swallowing and would be unable to tolerate peas, squash, and cooked carrots. Such foods would place a client with stage I dysphagia at risk for aspiration and choking. These foods are more conducive to a client with stage III dysphagia who is at the point of beginning to control the food in their mouth and is capable of tolerating various food textures. Choice D is incorrect. A client with stage I dysphagia has severe difficulty swallowing and would be unable to tolerate pies, cakes, and ice cream. Such foods would place a client with stage I dysphagia at risk for aspiration and choking. These foods are more conducive to a client with stage III dysphagia who is at the point of beginning to control the food in their mouth and is capable of tolerating various food textures. Learning Objective Identify the need for a client with stage I dysphagia to consume a puréed diet in order to reduce the risk of aspiration.

The nurse is reviewing laboratory data for a client taking prescribed phenytoin. The client's phenytoin level is 12 mcg/mL. Which action should the nurse take next? A. Evaluate the client for non-adherence. B. Instruct the client to skip the next scheduled dose. C. Assess the client for phenytoin toxicity. D. Document the result as within normal limits.

Explanation Choice D is correct. The therapeutic phenytoin level is 10-20 mcg/mL. A phenytoin level of 12 mcg/mL is considered therapeutic. Choices A, B, and C are incorrect. The phenytoin level is normal and does not require the client to be evaluated for non-adherence. Further, skipping the next dose and assessing the client for toxicity would be inappropriate because the level is therapeutic. Additional Info Phenytoin is indicated in the prevention of seizures. Phenytoin's most common adverse effects include gingival hyperplasia, excessive body hair growth, and folic acid deficiency. The normal phenytoin level is 10-20 mcg/mL, and toxicity is any level greater than 40 mcg/mL. Features of phenytoin toxicity include nystagmus, unsteady gait, and a worsening mental status as the levels rise.

The nurse observes a client with dementia not recognizing their family member. The nurse understands that this client is demonstrating signs of which of the following? A. Apraxia B. Agraphia C. Agnosia D. Aphasia

Explanation Choice C is correct. Agnosia is a clinical feature associated with dementia. Agnosia is the inability to identify familiar objects or people, even a spouse. Choices A, B, and D are incorrect. Apraxia is a clinical feature of dementia, and apraxia is the inability to perform familiar and purposeful tasks. An example of this is when a client is instructed to brush their hair or tie their shoes, they cannot perform the task. Agraphia is a term describing when a client has difficulty writing. This is often seen in early Alzheimer's disease. Aphasia is the difficulty finding the correct word, then is reduced to a few words, and finally is reduced to babbling or mutism. NCLEX Category: Physiological Adaptation Related Content: Alterations in Body Systems Question type: Knowledge/comprehension Additional Info Alzheimer's disease (AD) is the most common type of dementia. AD is commonly characterized by the progressive deterioration of cognitive functioning. Initial deterioration may be so subtle and insidious that others may not notice it. In the early stages of the disease, the affected person may be able to compensate for and hide cognitive deficits. Symptoms of AD include memory impairment that is progressive coupled with difficulty in executive functioning.

The nurse is caring for a client following cervical spinal surgery. Which of the following assessments would require follow-up? A. Active range of motion in both arms B. Scant drainage on the dressing C. Difficulty swallowing liquids D. Soreness at the operative site

Explanation Choice C is correct. Difficulty swallowing liquids indicates nerve damage that requires immediate follow-up. Following cervical spinal surgery, the client is likely placed in a cervical collar for a prescribed period. Manifestations that need to be reported following cervical spinal surgery include numbness and tingling in the upper extremities, difficulty swallowing, decreased motor strength, and respiratory depression. Choices A, B, and D are incorrect. These findings after this procedure are normal and do not require follow-up. The nurse would be concerned if active range of motion would be reduced, and scant drainage on the dressing does not concern for any hemorrhage. Following this procedure, it is likely that the client would experience soreness at the operative site. Additional Info The upper cervical spinal nerves innervate the diaphragm to control breathing. Monitor all clients following cervical spinal cord surgery for any reduction in respiratory rate. Additionally, if a client has difficulty swallowing, diminished sensation in the extremities, and movement in the upper extremities, this should be reported.

The nurse is caring for a client experiencing an episode of vertigo. The nurse should plan to take which essential action? A. Avoid sudden movement changes B. Provide additional pillows to support the client's head C. Raise the upper side rails of the bed D. Instruct the client to move the head slowly

Explanation Choice C is correct. Many actions should be taken for a client experiencing vertigo, but protecting the client's safety is essential. If a client is experiencing vertigo, this raises the risk of a fall. Interventions to prioritize include adequate lighting to the bathroom, raising the upper side rails on the bed, and providing the client with the call bell coupled with instructing the client to use it before getting out of bed. Choices A, B, and D are incorrect. These are appropriate actions for a client experiencing vertigo; however, they do not prioritize the client's safety. Additional Info Vertigo is characterized as a sense of whirling or turning in space. Many conditions may cause vertigo, including dehydration and inner ear disorders. A client with vertigo has a significant fall risk, and the nurse should mitigate this risk with fall precautions and frequent reinforcement to call for assistance before the client gets out of bed. Interventions for a client experiencing vertigo include - Fall precautions Propping the client's head up with additional pillows (may decrease the sensation) Avoid any sudden or jerky movements, especially with the head Administer medications, such as anticholinergics, as prescribed

A client with Alzheimer's disease is eating in the dining hall along with the other clients. Thirty minutes later, he says to the nurse, "When can I have my breakfast? They haven't given me anything to eat yet." The most appropriate response for the nurse would be: A. "I saw you eating breakfast 30 minutes ago." B. "Are you still not full? I'll ask the kitchen what they served you." C. "I can get you some bread if you like. What else would you like?" D. "You have to wait until it's lunchtime."

Explanation Choice C is correct. The client is displaying acute confusion. The best response for the nurse would be to provide the client with additional food as he requests it. Choice A is incorrect. The client with acute confusion can forget that he has already eaten. There is no use in arguing with the client. Choice B is incorrect. The client with acute confusion can forget that he has already eaten. There is no use in arguing with the client. Choice D is incorrect. The statement dismisses the client's concern. This is an inappropriate response.

The client has just been given an IV dose of morphine 6 mg for neuropathic pain. A few minutes later, the nurse notes that the client's respirations are now 8, and his blood pressure has dropped from 122/83 mmHg to 88/67 mmHg. Which nursing action is the most appropriate? A. Prepare for intubation B. Prepare to administer a dopamine infusion C. Administer naloxone D. Start an IV infusion of normal saline

Explanation Choice C is correct. The client is suffering from morphine toxicity. The nurse needs to administer the antidote, which is naloxone (Narcan). Choice A is incorrect. The client is in morphine toxicity. The nurse needs to administer an antidote to reverse the symptoms of respiratory depression. Preparing for intubation should not be the nurse's initial action. Choice B is incorrect. The drop in blood pressure is a result of morphine toxicity. Dopamine infusion is not yet necessary. Choice D is incorrect. Starting an IV infusion may be necessary; however, the first action of the nurse in case of this situation is to administer an antidote to morphine. Last Updated - 13, Oct 2021

A 76-year-old woman with dementia lives in an assisted living facility and often asks, "When will my sister come to visit me this afternoon?" The sister passed away last year. Which is the best response from the nurse? A. "This is so sad. I'm sorry to tell you but your sister died last year." B. "She won't be coming to visit today." C. "I understand you want her to visit you. Where did you and your sister grow up?" D. "Wait and see if she comes to visit today."

Explanation Choice C is correct. This statement shows compassion toward the patient. Asking where the client and her sister grew up allows her to think about her sister and reminisce without triggering anxiety or agitation. When communicating with a patient who has altered mental status, such as those with dementia, it is essential to foster therapeutic communication. Any statement that may trigger agitation or begin the grieving process should be avoided. Choices A and B are incorrect. These statements may trigger agitation or start the grieving process over again, which can be distressing to the patient. Choice D is incorrect. Saying the sister will not visit or that the patient should "wait and see" gives false hope and is deceptive. NCSBN Client Need Topic: Psychosocial Integrity, Subtopic: Therapeutic Communication Last Updated - 29, Oct 2021

Which of the following is an appropriate outcome for a client who is adversely affected with an impairment of their 2nd cranial nerve? A. The client will not experience sensory overload in the hospital. B. The client will list ways to effectively decrease their blood pressure. C. The client will participate in physical therapy to improve balance. D. The client will remain free of falls despite 2nd cranial nerve impairment.

Explanation Choice D is correct. "The client will remain free of falls despite 2nd cranial nerve impairment" is an appropriate client outcome for a client who is adversely affected with an impairment of their 2nd cranial nerve. This client has a visual deficit because the 2nd cranial nerve is the optic nerve; visual deficits place clients at risk for falls. Choice A is incorrect. "The client will not experience sensory overload in the hospital" is not an appropriate client outcome for a client who is adversely affected with an impairment of their 2nd cranial nerve. This client has a visual deficit because the 2nd cranial nerve is the optic nerve and visual deficits place clients at risk for sensory deprivation in the hospital, rather than sensory overload. Choice B is incorrect. "The client will list ways to effectively decrease their blood pressure" is not an appropriate client outcome for a client who is adversely affected with an impairment of their 2nd cranial nerve. This client has a visual deficit because the 2nd cranial nerve is the optic nerve and visual deficits are not associated with hypertension. Choice C is incorrect. "The client will participate in physical therapy to improve balance" is not an appropriate client outcome for a client who is adversely affected with an impairment of their 2nd cranial nerve. This client has a visual deficit because the 2nd cranial nerve is the optic nerve and visual deficits are not corrected with physical therapy, but instead with low vision specialists and other members of the ophthalmology team. Last Updated - 28, Jan 2022

The nurse is teaching a client with Parkinson's disease about dietary considerations. The nurse understands that this client is at highest risk for A. constipation and drooling. B. drooling and a loss of appetite. C. loose stools and choking. D. dysphagia and aspiration.

Explanation Choice D is correct. A common symptom often seen in Parkinson's clients is dysphagia resulting from esophageal dysmotility. More specifically, dysphagia places the client at an increased risk of aspiration, especially as the disease progresses. As time progresses, swallowing becomes progressively difficult, and the risk of acquiring aspiration pneumonia increases. Choice A is incorrect. Although drooling is a symptom of Parkinson's disease, drooling does not take priority over dysphagia and the associated increased risk of aspiration. Constipation may also occur in clients with Parkinson's disease due to a slowing of the motility of the gastrointestinal tract, lack of adequate nutritional and fluid intake, and/or impaired mobility. Choice B is incorrect. Drooling is a symptom of Parkinson's disease, typically occurring in the latter stages of the disease. A loss of appetite (anorexia) is not a problem commonly associated with Parkinson's disease. Choice C is incorrect. Although dysphagia places Parkinson's disease clients at an increased risk of aspiration, diarrhea is not a common concern in clients with Parkinson's disease. Therefore, while a portion of this answer is correct, there is a better answer than Choice C. Learning Objective During a client and family education session on Parkinson's disease dietary considerations, dysphagia and the associated risk of aspiration should be the primary focus. Additional Info Parkinson's disease is a slowly progressive, degenerative disorder characterized by resting tremors, stiffness (rigidity), slow and decreased movement (bradykinesia), and eventually gait and/or postural instability. Parkinson's disease is separated into stages according to the symptoms and degree of disability. Stage 1 is mild disease with unilateral limb involvement, whereas the client with stage 5 is entirely dependent on all activities of daily living (ADLs). Other classifications refer simply to mild, moderate, and severe disease.

The nurse is caring for a client who has had an exacerbation of Bell's palsy. The client is experiencing paralysis of their eye, the nurse should plan to A. tape an eye patch to the affected eyelid at all times. B. instruct the client to keep both eyes closed. C. assess the pupil's size and reactivity to light. D. apply the prescribed ocular lubricant to the affected eye.

Explanation Choice D is correct. Bell's palsy is a lower motor neuron facial nerve palsy that can result in the weakness of facial muscles and the muscles responsible for eye closure (orbicularis oculi). A client with Bell's palsy who cannot blink would be unable to close the affected eye. As a result, the cornea becomes overly dry, leading to an increased risk of corneal ulceration and scarring. Eye lubricant (i.e., typically artificial tears) must be applied as often as every hour during the day to keep the eye moist and prevent corneal drying. A moisturizing eye ointment may be used at night. Choices A, B, and C are incorrect. Applying an eye patch with tape on the eyelid may cause the patch to slip into the open eye and cause a corneal abrasion. During the day, the client should protect the open eye with glasses or goggles. At night, the client may use a soft eye patch to cover the open eye, but it should not be taped to the eyelid. Instead, the soft eye pad should be secured with one end of the tape on the client's forehead and the other end on the cheek diagonally. It is not necessary for the client to keep the unaffected eye closed. Bell's palsy does not affect the pupil's reaction to light and accommodation. Learning Objective Recognize that when caring for a client with Bell's palsy, the plan of care should include an order from the health care provider (HCP) to apply eye lubricant to prevent corneal drying.

The nurse is assessing a client who is postoperative following a hypophysectomy. Which of the following findings should the nurse report to the primary healthcare provider (PHCP) immediately? A. Client reports a decreased smell B. No bowel movement in two days C. Foul-smelling breath D. Hourly urine output of 125 mL

Explanation Choice D is correct. Following hypophysectomy, the client is at risk of developing diabetes insipidus (DI). An hourly urine output of 125 mL would be considered polyuria because, in 24 hours, that would equate to 3000 mL. Choices A, B, and C are incorrect. A decreased smell is an expected finding after hypophysectomy. The client may experience this finding for a few months. Constipation is concerning because the client should be instructed not to strain for a bowel movement as this may increase intracranial pressure. The client is instructed not to brush their teeth following this procedure and should floss and use mouthwash as an alternative. Thus, incomplete oral care may produce foul-smelling breath. NCLEX Category: Physiological Adaptation Activity Statement: Illness management Question type: Analysis Additional Info After hypophysectomy, the client should be monitored closely for increased intracranial pressure, headaches, urine output, and vital signs. The client should be instructed to avoid blowing their nose, coughing, or straining. The most serious adverse effect of this surgery is CSF leakage, increased intracranial pressure, infection, and diabetes insipidus. Last Updated - 29, Apr 2022

The nurse is assessing a client with Guillain Barré syndrome. Which of the following would be an expected finding? A. Hyperreflexia B. Perseveration C. Dystonia D. Paresthesia

Explanation Choice D is correct. Guillain Barré is a polyneuropathy that is manifested by paralysis, paresthesia, autonomic disturbances, and depressed or absent reflexes. The paresthesia is typically found in the peripheral extremities and may persist for quite some time, even after the return of motor function. Choices A, B, and C are incorrect. Depressed or absent reflexes are a hallmark of this disease process. Perseveration is when the individual continues to repeat something, such as a phrase. This is commonly seen in Alzheimer's disease. This is not a feature in Guillain-Barré. Dystonia is an adverse effect associated with dopaminergic drugs such as haloperidol. This muscle repetitive muscle contraction is not related to Guillain Barré. Additional information: The nurse should recognize Guillain Barré quickly and ensure a patent airway, as the ascending paralysis may impact the diaphragm. The cause of Guillain Barré can be certain pathogens such as Campylobacter jejuni, which may induce massive peripheral nerve demyelination. Other causes include certain immunizations and bone marrow transplantation. Additional Info As a competent adult, it is the client's right to refuse treatment for any reason, even when refusal might compromise the client's health or death. The nurse should inquire about a refusal and clear up any questions the client may have regarding the medication or treatment. The nurse should also educate the client on the purpose of the prescription and treatment. If that is ineffective, the nurse should contact the PHCP and relay the refusal. All actions, including the client's statements regarding the refusal, should be documented.

A 30-year-old man was involved in a head-on collision and was unconscious for two minutes prior to EMS arrival. Five minutes before arriving to the hospital, the paramedic notices clear fluid draining from the patient's nose. Having seen this before, the paramedic places a drop from the patient's nose onto a piece of gauze. The nurse is looking for a clinical finding that is called the halo's sign. What type of fracture does the paramedic suspect the patient has? A. Depressed skull fracture B. Traumatic linear skull fracture C. Subarachnoid hemorrhage D. Basilar skull fracture

Explanation Choice D is correct. Halo's sign is an indication of a basilar skull fracture. Rhinorrhea can occur from a basilar skull fracture. When this finding is assessed, the provider can place a drop from the nose onto a piece of gauze. The CSF will form a ring around the outside of the drop. This is halo's sign. Choices A, B, and C are incorrect. Halo's sign is clinically linked to a basilar skull fracture. Although halo's sign can sometimes occur because of a depressed or linear skull fracture, it is not likely. Halo's sign is almost always an indicator of a basilar skull fracture. A CSF leak occurs in about 20% of patients after suffering from a basilar skull fracture. This occurs because of a break in the temporal bones of the skull, which are the bones that are most commonly broken. CSF fluid can leak through the subarachnoid space after the destruction of the meningeal structure. NCSBN Client Need Topic: Physiological Adaptation, Sub-topic: Alterations in Body Systems, Peripheral Nerve and Spinal Cord Problems Last Updated - 16, Nov 2021

Minimizing and challenging the client's report of pain and pain intensity is: A. Often necessary if the client has a history of substance abuse. B. Often necessary if the client has a history of drug seeking behavior. C. Contrary to and in violation of the Nightingale oath. D. Contrary to and in violation of the American Nurses Association's standard of care.

Explanation Choice D is correct. Minimizing and challenging the client's report of pain/pain intensity is in violation of the American Nurses Association's standards of care about pain/pain management. Specifically, the American Nurses Association's Standards of Professional Performance for Pain Management Nursing. For example, nurses are mandated to document pain as expressed by the client regardless of what the nurse believes to be true and accurate. Choice A is incorrect. Minimizing and challenging the client's report of pain/pain intensity is not often necessary if the client has a history of substance abuse; this expression and reporting of pain must be considered valid and accurate. Nurses are mandated to document pain as expressed by the client regardless of what the nurse believes to be accurate. Choice B is incorrect. Minimizing and challenging the client's report of pain/pain intensity is not often necessary if the client has a history of drug-seeking behavior; this expression and reporting of pain must be considered valid and accurate. Nurses are mandated to document pain as expressed by the client regardless of what the nurse believes to be accurate. Choice C is incorrect. Minimizing and challenging the client's report of pain/pain intensity is not in violation of the Nightingale oath. There is no mention of pain management in the Nightingale oath. Last Updated - 02, Feb 2022

The nurse reviews a client's medical history and identifies a diagnosis of presbycusis. The nurse should integrate which intervention in the care plan? A. Have educational materials in large print B. Provide an eye patch to the affected eye C. Request food be seasoned with herbs D. Move closer to the better-hearing ear

Explanation Choice D is correct. Presbycusis is a type of sensorineural hearing loss associated with aging. Sensorineural hearing loss is often permanent. Interventions for a client with this type of hearing loss include speaking in the ear less affected, speak clearly and slowly, avoid shouting, and ensure that the environment is well lit while conversing. Choices A, B, and C are incorrect. Presbycusis is a type of sensorineural hearing loss, and interventions such as scanning the room, having large print for reading materials, and seasoning food are not relevant to this condition. Scanning the room would be an intervention appropriate for visual field loss. Additional Info Hearing loss is divided into sensorineural or conductive. Conductive hearing loss is caused by obstruction. Causes of this type of hearing loss include cerumen, foreign body, water, edema, infection, or tumor. This type of hearing loss may be reversible. Impairments of the nerve fibers cause sensorineural hearing loss. Causes of this type of hearing loss include prolonged exposure to noise, ototoxic substances (aminoglycosides), diabetes mellitus, and presbycusis (age-related hearing loss). This type of hearing loss is often not reversible.

In preparing for the admission of a toddler who has been diagnosed with febrile seizures, which of the following is the most important nursing action? A. Order a stat admission CBC. B. Place a urine collection bag and specimen cup at the bedside. C. Place a cooling mattress on his bed. D. Pad the side rails of his bed.

Explanation Choice D is correct. The child has a diagnosis of febrile seizures. Precautions to prevent injury and promote safety should take precedence. Children between 6 months and five years of age are at a higher risk for fever-induced (febrile) seizures. Febrile seizures are not associated with neurological seizure disorders. The priority in nursing care for a patient (of any age) who has experienced a seizure is to implement safety precautions that decrease the likelihood of injury if/when another seizure occurs. Choice A is incorrect. Only a physician can order lab work. Choice B is incorrect. Preparing for routine laboratory studies is not as high of a priority as preventing injury and promoting safety. Choice C is incorrect. A cooling blanket must be ordered by the physician and is usually not used unless other methods for the reduction of fever have not been successful. NCSBN Client Need Topic: Safe and Effective Care Environment, Subtopic: Safety and Infection Control, Seizure Precautions Last Updated - 07, Feb 2022

The nurse is discharging the client that has been admitted due to subarachnoid hemorrhage. The client still has some speech and balance deficits. Which referral should the nurse make? A. Refer the client to hospice care. B. Refer the client to speech therapy. C. Refer the client to physical therapy. D. Refer the client to a home health agency. Submit Answer

Explanation Choice D is correct. The client is going home, thus the client needs to be referred to a home health agency so that there is continuity of care even at home. Choice A is incorrect. Hospice care is for clients that are terminally ill. This client is not terminally ill. Choice B is incorrect. Speech therapy aids clients in regaining speech and swallowing abilities. Speech therapy should have been initiated and ongoing while the client was in the hospital. Choice C is incorrect. Physical therapy aids clients in regaining muscle strength and balance. Physical therapy should have been initiated and ongoing while the client was in the hospital. Last Updated - 18, Jan 2022

The nurse is caring for a post-stroke client when suddenly she notes that the client has a fixed and dilated pupil. What would be the most appropriate action by the nurse? A. Reduce environmental stimuli. B. Reassess after ten minutes. C. Check the client's blood pressure. D. Notify the physician.

Explanation Choice D is correct. The client is manifesting signs of increased intracranial pressure. This situation warrants immediate medical intervention to decrease the ICP. The nurse needs to notify the physician immediately. Choice A is incorrect. A fixed and dilated pupil signifies an increase in ICP. Reducing environmental stimuli is not an appropriate intervention at the time. Choice B is incorrect. There is no need to reassess after ten minutes as this warrants immediate attention from the healthcare team. Choice C is incorrect. Checking the client's blood pressure is unnecessary.

Which transcultural difference would you apply to your pain management practice? A. The fact that Native Americans may tend to refuse pain medication and also view pain as the will of God. They tend to be silent about their pain and may also have a higher pain tolerance than other ethnicities and cultures. B. The fact that Mexican Americans may tend to believe that pain is a signal that life will end and that health will never return. C. The fact that Filipino Americans may tend to be silent about their pain and they may also have a higher pain tolerance than other ethnicities and cultures. D. The fact that Arab Americans may tend to be private about communicating their level of pain with people outside of their immediate family.

Explanation Choice D is correct. The fact that Arab Americans may tend to be private about communicating their level of pain with people outside of their immediate family is a transcultural difference that you would apply to your pain management practice. Other transcultural differences for other cultures and ethnicities are as follows: Native Americans may tend to be silent about their pain and they may also have a higher tolerance for pain than other ethnicities and cultures. Mexican Americans may tend to believe that pain is a natural part of life and that pain is an indication of the gravity of one's disease or condition. Filipino Americans may tend to refuse pain medication and also view pain as the will of God. Choice A is incorrect. The fact that Native Americans may tend to refuse pain medication and also view pain as the will of God is NOT accurate so you would NOT apply this knowledge into your pain management practice. On the other hand, you would apply the fact that Native Americans may tend to be silent about their pain and they may also have a higher tolerance for pain than other ethnicities and cultures. Choice B is incorrect. The fact that Mexican Americans may tend to believe that pain is a signal that life will end and that health will never return is NOT accurate so you would NOT apply this knowledge into your pain management practice. On the other hand, you would apply the fact that Mexican Americans may tend to believe that pain is a natural part of life and that pain is an indication of the gravity of one's disease or condition. Choice C is incorrect. The fact that Filipino Americans may tend to be silent about their pain and they may also have a higher tolerance for pain than other ethnicities and cultures is NOT accurate so you would NOT apply this knowledge into your pain management practice. On the other hand, you would apply the fact that Filipino Americans may tend to refuse pain medication and they also view pain as the will of God. Last Updated - 13, Jan 2022

The client in the intensive care unit is admitted because of a subdural hematoma. The nurses are currently in the middle of a clinical hand-off when suddenly the alarm goes off, indicating a drop in the client's blood pressure. The initial action of the nurses would be: A. Turn the alarm off and inform the incoming nurse of the decrease in blood pressure. B. Lower the blood pressure alarm limits on the monitor during the hand-off. C. Continue with the hand-off and include the drop in blood pressure in the report. D. Assess the client and then resume the hand-off.

Explanation Choice D is correct. The nurse should assess the patient first and provide any appropriate interventions to address the client's situation. Once the client's needs are met, other tasks can be performed. Choice A is incorrect. Alarms should not be turned off. A signal indicating decreased blood pressure requires immediate assessment. Choice B is incorrect. Blood pressure alarm limits should not be changed unless ordered by the physician. Choice C is incorrect. The patient should be assessed immediately as the patient's well-being holds priority over the convenience of the nurse.

Which statement below relating to pain and pain perception is accurate? A. Allodynia is the pathophysiological absence of pain when a painful stimulus is applied. B. Scientific evidence does not support the presence of pain during neonatal circumcision. C. Hyperanalgesia is the opposite of hyperpathia, both of which are abnormal pain responses. D. The perception of pain and its impact on our clients greatly varies among people.

Explanation Choice D is correct. The perception of pain and its impact on our clients greatly varies among people. For example, gender, cultural beliefs, and individuals' unique pain thresholds all impact our clients' perceptions of pain. Choice A is incorrect. Allodynia is the pathophysiological perception of pain when no painful stimulus is applied. Allodynia, like other abnormal pain processing and pain perception processes, indicates the presence of a neuropathic process. Choice B is incorrect. Scientific evidence supports the presence of pain during neonatal circumcision, something that was not recognized in the past. Choice C is incorrect. Hyperalgesia is a synonym for hyperpathia. Hyperalgesia is an abnormal pain response that is characterized by an intense and severe perception of pain when the stimulus is not at all severe. Last Updated - 15, Feb 2022

The nurse is assessing a patient with suspected neurological issues. The patient's speech is delivered with normal rhythm but filled with words that do not form any meaningful statements. The patient is also unable to write or repeat back words and does not appear to understand the nurse's instructions or questions. The nurse would recognize these symptoms as: A. Broca's aphasia B. Global aphasia C. Expressive aphasia D. Wernicke's aphasia

Explanation Choice D is correct. This patient is showing symptoms consistent with Wernicke's aphasia, which refers to a lesion in the left posterior superior temporal lobe/language area of the brain. It is characterized by the ability to produce verbal language but mix similar sounding words so that speech is often incomprehensible. Reading, writing, oral comprehension, and repetition are affected. Choice A is incorrect. Broca's aphasia refers to a lesion in the anterior language area of the brain (motor speech cortex). Verbal comprehension remains intact, but the patient is usually unable to form words at all and has difficulty with writing and repetition. Choice B is incorrect. Global aphasia is the most common and severe type of aphasia due to a large lesion that damages both the anterior and posterior language areas of the brain. Speech, comprehension, repetition, reading, and writing are absent or severely impaired in global aphasia. Choice C is incorrect. Expressive aphasia describes the same deficits as noted above for Broca's aphasia.

The nurse is evaluating a patient's response to peripheral pain. Which technique should the nurse use to perform this evaluation? A. Pressure on the patient's mid-back B. Sternal rub C. Squeezing the sternocleidomastoid muscle D. Pressing on the patient's nail bed

Explanation Choice D is correct. To test peripheral responses to pain, health care providers should apply pressure to outer body parts such as the toes or fingers. Pressing on the patient's nail bed is the most appropriate action. Choice A is incorrect. Applying pressuring on the patient's mid-back does not evaluate peripheral pain. Choice B is incorrect. Sternal rubs are most often used to test consciousness. Choice C is incorrect. Squeezing, the patient's sternocleidomastoid muscle does not evaluate peripheral pain. NCSBN client need Topic: Pharmacological Integrity, Reduction of Risk Potential Last Updated - 11, Jan 2022

You are working with older adults in the clinic. The 80-year-old woman is brought to the clinic by her family with fever and changes in her mental status. When attempting to differentiate between delirium and dementia, you know that delirium is characterized by which of the following? Select all that apply. A. Abrupt onset B. Change in psychomotor activity C. Irreversible D. Lasts for months to years

Explanation Choices A and B are correct. An acute illness (fever, sepsis, infection) typically causes delirium, so delirium often has an abrupt onset (Choice A) with rapid progression. There are significant changes in activity resulting in hyperactivity or hypoactivity (Choice B). Delirium is typically reversible when the underlying illness is resolved. Delirium typically lasts for hours to days, whereas dementia lasts for months to years and is usually irreversible. Choices C and D are incorrect. These two characterize dementia. Dementia has a variety of causes with gradual changes in mentation. In dementia, psychomotor changes occur later in the disease; speech is sparse and may progress to mutism as the disease advances. NCSBN Client Need Topic: Physiological Adaptation; Sub-topic: Alterations in Body Systems

The nurse is developing a plan of care for a client with an impairment to the hypoglossal cranial nerve. Which of the following should the nurse include in the client's plan of care? Select all that apply. A. Observe the client during meals B. Keep suction at the bedside C. Provide large print education materials D. Teach the client to scan the room E. Alternate the use of an eye patch

Explanation Choices A and B are correct. The hypoglossal cranial nerve (XII) is central to the skeletal muscles of the tongue and assists with swallowing. If a client has an impairment of this cranial nerve, aspiration precautions should be implemented. These precautions include observing the client during meals and having patent suction at the bedside. Choices C, D, and E are incorrect. The hypoglossal nerve has no relevance in vision. Thus, providing large print education materials, teaching the client to scan the room, and alternating the use of an eye patch are not relevant to this cranial nerve. An eye patch is an effective intervention for a client experiencing diplopia. Additional Info Aspiration precautions involve collaborating with the registered dietician and/or speech therapist to determine which would be appropriate. In general, aspiration precautions involve supervised mealtimes, having the head of the bed elevated during a meal, thickening the liquids as directed, and coaching the client through swallowing. If the nurse is feeding a client with aspiration precautions, the nurse should not mix any food consistencies and always have oral suction available. Last Updated - 14, Nov 2022

he nurse is caring for a client receiving lorazepam. Which of the following reported herbal supplements would require follow-up? Select all that apply. A. Kava B. Glucosamine C. Valerian D. Garlic E. Saw palmetto

Explanation Choices A and C are correct. Lorazepam is a CNS depressant, and the client should avoid potentiating the effects of this medication. Herbal products such as kava and valerian are CNS depressant medications that should not be given concurrently while a client is receiving lorazepam. Lorazepam and one of these medications may cause profound sedation. Choices B, D, and E are incorrect. Glucosamine is an herbal product that may benefit clients with osteoarthritis in the knees, waist, and hips. This medication does not cause CNS depression. Garlic may be taken to assist a client in reducing their cholesterol and should be avoided if the client is taking anticoagulants. This medication does not alter the CNS. Saw palmetto may be taken for men who have prostate hyperplasia. This herbal supplement does not alter the CNS. Additional Info Herbal products may cause serious interactions with prescribed medications. CNS depressant medications such as lorazepam should not be taken concurrently with herbal products that may cause CNS depression. The nurse should always obtain a comprehensive list of a client's medications, including over-the-counter (OTC) products.

The nurse is caring for a client with an acute exacerbation of Bell's palsy. Which of the following prescriptions would the nurse anticipate? Select all that apply. A. Prednisone B. Donepezil C. Pyridostigmine D. Valacyclovir E. Topiramate

Explanation Choices A and D are correct. Bell's palsy classically causes facial nerve paralysis. It is usually idiopathic. However, etiologies such as herpes simplex virus may be present. Exacerbations of Bell's palsy are treated with corticosteroids (prednisone, choice A) and antivirals (valacyclovir, choice D). Corticosteroids decrease facial nerve inflammation, and antivirals address the possible underlying viral etiology. Choices B, C, and E are incorrect. Donepezil is an acetylcholinesterase inhibitor utilized to manage dementia in Alzheimer's disease (choice B). Although Donepezil does not decrease the progression of Alzheimer's disease, it does help symptoms by improving cognition and behavior. Pyridostigmine is an acetylcholinesterase inhibitor indicated for improving muscle strength in myasthenia gravis (choice C). Topiramate is an anticonvulsant indicated for epilepsy and migraine headache prevention (choice E). Learning Objective Understand that antivirals and corticosteroid medications are frequently used in managing Bell's palsy Additional Info Exacerbations of Bell's palsy usually occur abruptly with unilateral facial paralysis. This is accompanied by eyebrow sagging, diminished taste, decreased eye tearing, and drooping of the mouth on the affected side. Nursing care is aimed at mitigating symptoms by using artificial tears and ointment. A client may also use an eye patch on the affected eye at night.

The nurse is caring for a client diagnosed with epilepsy. The nurse should anticipate a prescription for which of the following medications? Select all that apply. A. Topiramate B. Risperidone C. Prazosin D. Hydroxyzine E. Lorazepam

Explanation Choices A and E are correct. Epilepsy is an idiopathic condition that requires maintenance treatment by using anticonvulsants. Topiramate is an anticonvulsant that may be used in the prevention of seizures. Lorazepam is also indicated in epilepsy in the event of a patient experiencing an acute seizure. The topiramate should be used for maintenance purposes, and the lorazepam would be indicated for an acute seizure. Choices B, C, and D are incorrect. Risperidone is indicated for psychotic disorders such as schizophrenia. Prazosin is an antihypertensive that may be used for high blood pressure. This medication also may be indicated for psychiatric illnesses such as PTSD. Hydroxyzine is indicated for anxiety disorders as well as allergic rhinitis. Epilepsy is an idiopathic condition that requires management with anticonvulsants such as topiramate, valproic acid, or phenytoin. Acute seizures are managed with benzodiazepines such as lorazepam or diazepam. These medications work to terminate a seizure. During an acute seizure, the nurse should place the patient on their side, loosen restrictive clothing, and anticipate a prescription for a parenteral benzodiazepine such as diazepam.

The nurse is caring for a client with a migraine headache. Which assessment findings should the nurse expect? Select all that apply. A. Unilateral frontotemporal pain B. Drowsiness C. Phonophobia D. Shuffling gait E. Dysphagia F. Vomiting

Explanation Choices A, B, C, and F are correct. An array of symptoms may be reported for a client experiencing a migraine headache (MH). The most common manifestations associated with an acute migraine headache include Unilateral frontotemporal pain that may be described as throbbing or dull Sensitivity to light (photophobia) and sound (phonophobia) Nausea and/or vomiting Altered mentation (drowsiness) Dizziness, numbness, and tingling sensations Choices D and E are incorrect. An acute migraine headache (MH) would not produce symptoms such as a shuffling gait. This clinical feature would be linked to Parkinson's disease. Dysphagia is not a manifestation associated with an MH. Additional Info Migraine headaches have a complex pathophysiology that is not entirely understood. The current thought process regarding this syndrome is that it is caused by a combination of neuronal hyperexcitability and vascular, genetic, hormonal, and environmental factors. During an acute migraine headache, often the client may feel as though they are experiencing a stroke because of transient facial paralysis and/or numbness that may be experienced.

You are working in the emergency department when a patient with a suspected stroke arrives. According to the American Heart Association (AHA), the general immediate assessment and stabilization should include: Select all that apply. A. Activate the stroke team B. Check and treat the glucose C. Order an immediate CT or MRI of the brain D. Administer rtPA

Explanation Choices A, B, and C are correct. According to the AHA, the immediate general assessment and stabilization should include: assess the ABCs and vital signs, provide oxygen as needed, obtain an IV, check glucose and treat as needed, perform an essential neurologic screening, activation of the stroke team, order an immediate CT or MRI of the brain, and obtain an ECG. All of these actions should be included within the first 10 minutes after arrival at the ED. Choice D is incorrect. The decision of whether or not to give rtPA will depend on the results of the CT scan or MRI. If the provider determines that there is no brain hemorrhage, the team should complete the fibrinolytic checklist before deciding whether or not to give rtPA. NCSBN Client Need Topic: Management of Care, Sub-Topic: Establishing Priorities, Neurologic; Prioritization

The nurse is caring for a client who was prescribed carbidopa and levodopa for Parkinson's disease. The nurse should instruct the client that this medication may cause Select all that apply A. urine to appear darker. B. hallucinations. C. dizziness upon standing. D. dry, non-productive cough. E. painful rash that spreads and blisters.

Explanation Choices A, B, and C are correct. Levodopa-carbidopa is the mainstay treatment for Parkinson's disease. The medication helps with the movement symptoms but has an array of adverse effects, including psychiatric symptoms that may cause the client to experience nightmares, paranoia, sleep disturbances, and bouts of psychosis. This is because the medication increases dopamine levels causing these psychiatric symptoms. Orthostatic hypotension is a significant concern with this medication because of alterations in the baroreceptors. This effect is intensified if the client is on anti-hypertensives. Choices D and E are incorrect. A dry, non-productive cough is not associated with this medication. Instead, this would be found in clients prescribed an ACE inhibitor. Steven-Johnson syndrome is not related to this medication which is described as a painful rash that spreads and blisters. Additional Info Levodopa-carbidopa is a mainstay treatment for Parkinson's disease ➢ The nurse should educate the client to consume their dietary protein throughout the day to avoid fluctuations in levodopa absorption. A high-protein diet may reduce the effectiveness of the medication. ➢ This medication should be administered on time to avoid significant fluctuations in dopamine which may intensify symptoms. ➢ Darkening of secretions such as sweat, tears, and urine is common and harmless.

ou are taking care of an infant newly diagnosed with hydrocephalus. Which of the following assessment findings do you expect? Select all that apply. A. Increased head circumference B. Macewen's sign C. Sunken anterior fontanelle D. Setting sun eyes

Explanation Choices A, B, and D are correct. A is correct. The increased head circumference is due to increased CSF in the cranial vault due to impaired absorption within the subarachnoid space. This is often the first and most noticeable sign of hydrocephalus. B is correct. Macewen's sign is an indication of hydrocephalus. This sign is positive when the nurse percusses the skull bones and hears a 'cracked-pot' sound. This sound is due to thin, widely separated skull bones with hydrocephalus. D is correct. Setting sun eyes is an assessment finding found in children with hydrocephalus that has progressed so far, causing increased ICP. The child looks as if they are always looking down with more prominent sclera in the top part of their eyes. Choice C is incorrect. A sunken anterior fontanelle would indicate dehydration or low fluid volume status. In hydrocephalus, there is increased fluid volume in the head, not decreased. The nurse would see a tense, bulging fontanelle.

The nurse is developing a plan of care for a client who has epilepsy and is undergoing an electroencephalogram. Which of the following should the nurse include in the client's plan of care? Select all that apply. A. Provide padding to the side rails B. Verify suction is at bedside and working properly. C. Keep bite block at bedside in case of seizure. D. Ensure nasal cannula is available and working at the bedside. E. Establish peripheral vascular access

Explanation Choices A, B, and E are correct. Ensuring the side rails are raised and padded will provide a safe environment for the client in case of a seizure. It is imperative to have suction ready at the bedside should the client vomit during a seizure. Timely clearing of the airway will prevent aspiration, maintain a patent airway, and keep your client safe. Suctioning the client should only occur once the seizure has terminated, as it is contraindicated to putting objects in the client's mouth. Ensuring that peripheral vascular access is essential because if the client has a seizure, parenteral benzodiazepines (diazepam/lorazepam) are necessary. Choices C and D are incorrect. It is not appropriate to put a bite block or any other object into a client's mouth that is seizing. This could result in injury to yourself or the client. Nursing priorities during a seizure are ensuring the client is safe and has a patent airway. While it is essential to have oxygen available in the room, a nasal cannula is inappropriate for this client. There should be a face mask or Ambu bag readily available that is an appropriate size and connected to 10 L of 100% oxygen. Additional Info For the client with epilepsy, the nurse should maintain seizure precautions during the EEG. Seizure precautions include ensuring that oxygen and suctioning equipment with an airway are readily available. If the client does not have IV access, insert a saline lock, especially if he or she is at significant risk for generalized tonic-clonic seizures. The saline lock provides ready access if IV drug therapy must be given to stop the seizure. Last Updated - 18, Nov 2022

The nurse is teaching a group of students about contributing factors for delirium. The nurse is correct in identifying that delirium can be caused by: Select all that apply. A. Fever B. Alzheimer's disease C. Hypoglycemia D. Vascular disease E. Infection

Explanation Choices A, C, and E are correct. Delirium is an alteration in mental status that occurs abruptly. Delirium, unlike dementia, is reversible with treatment. Contributing factors for delirium include fever, hypoglycemia, and infection. Choices B and D are incorrect. Alzheimer's disease is a form of dementia, not delirium. Vascular disease (tiny cerebral infarcts) is a direct contributor to dementia. A preventative measure for vascular dementia is mitigation of hypertension and diabetes as these two conditions directly cause vascular dementia. Learning Objective Understand the common causes of delirium. Additional Info Delirium is an altered sensorium. It is characterized by acute changes in the patient's level of consciousness. Hyperactive delirium is characterized by agitation, restlessness, and emotional lability. Hypoactive delirium is characterized by flat affect, apathy, lethargy, or decreased responsiveness. Many causes of delirium include medications (dexamethasone, opioid toxicity), nicotine withdrawal, dehydration, uncontrolled pain, constipation, urinary retention, infection, hypoxia, renal failure, hyponatremia, hypercalcemia, hyperglycemia, and emotional distress. Initially, non-pharmacological interventions should be attempted to identify and address reversible etiology and relieve terminal agitation/delirium. For example, address the reversible causes such as treating constipation or discontinuing medications such as dexamethasone, modifying precipitating factors such as sensory deprivation or uncontrolled pain, etc. If no rapidly reversible factors are identified or if the patient is terminal, dopamine antagonists must be used.

The nurse is assessing a client who is suspected of having myasthenia gravis. Which of the following would be an expected finding? Select all that apply. A. Diplopia B. Butterfly rash C. Facial muscle weakness D. Shuffling gait E. Ptosis

Explanation Choices A, C, and E are correct. Key clinical features of myasthenia gravis (MG) include diplopia, ptosis, facial muscle weakness, and may progress to respiratory failure. Some of the earlier manifestations associated with MG are ocular. Choices B and D are incorrect. Shuffling gait is a classic manifestation associated with Parkinson's disease. A butterfly rash is a common dermatological finding associated with lupus. Additional Info Myasthenia gravis is a rare and serious autoimmune disorder that impairs the acetylcholine receptors. Commonly occurring more in women, this disorder impacts motor nerves which impair facial and eye muscles. In its severe form, myasthenia gravis may impact respiratory muscles causing respiratory failure. Last Updated - 17, Jul 2022

You are teaching a student nurse about various types of pain. The student nurse should realize which of the following types of pain are accurately paired with one of their signs or symptoms? Select all that apply. A. Chronic pain: The vital signs are normal. B. Chronic pain: The sympathetic nervous system is activated. C. Acute pain: The pulse, blood pressure, and respiratory rate are increased. D. Acute pain: The parasympathetic nervous system is activated. E. Somatic pain: A type of neuropathic pain. F. Somatic pain: Pain sensation originates from the bones, skin, and muscles. G. Visceral pain: A type of neuropathic pain. H. Visceral pain: The vital signs are normal

Explanation Choices A, C, and F are correct. Chronic Pain is characterized by typical vital signs (Choice A), whereas acute pain is characterized by increased pulse, blood pressure, and respiratory rate (Choice C). In chronic pain, pupils can be healthy or dilated, and the client can be withdrawn and depressed. In chronic pain, the parasympathetic nervous system is activated. In acute pain, the sympathetic nervous system is activated. Therefore, the presentation includes the features of sympathetic activation. Pulse, blood pressure, and respiratory rate are increased. The pupils are dilated; the client can be restless and show pain behaviors such as guarding the painful area and crying. Somatic pain originates from the bones, the skin, and the muscles (Choice F) and somatic pain is a type of nociceptive pain, rather than neuropathic pain. It is essential to understand the terminology of pain based on: Onset and duration (Acute pain vs. Chronic pain). Origin (Somatic pain vs. Visceral pain) - The fully functional nervous system transmits messages that a part of the body is damaged. Somatic pain occurs when the damage involves the bones, the skin, and the muscles. Visceral pain occurs when the injury involves the internal organs in the central cavities of the body (also called the viscera). Physical pain may be described as sickening, deep, or dull in quality. In visceral pain, vital signs are increased. Cause of the pain (Nociceptive vs. Neuropathic) Nociceptors are pain receptors present in many parts of the body, including internal organs. Nociceptive pain arises secondary to damage/injury caused to the body part by an external stimulus or condition. This is often acute but may also be chronic. Examples include burns, bee stings, stab wounds, tumors, inflammatory arthritis, etc. Both Somatic and Visceral pain are types of Nociceptive pain. Neuropathic pain is mediated by the nerves and is from damage to the nervous system itself. It may be because of injury secondary to the central or peripheral nervous system from different causes. Examples: Multiple sclerosis, peripheral neuropathy, etc. It may be stabbing, shooting, or aching in nature. This type of pain is often chronic. Choice B is incorrect. In chronic pain, the parasympathetic nervous system, rather than the sympathetic nervous system, is activated. Choice D is incorrect. In acute pain, the sympathetic nervous system, rather than the parasympathetic nervous system, is activated. Choice E is incorrect. Somatic pain is a type of nociceptive pain, not neuropathic pain. Choice G is incorrect. Visceral pain is a type of nociceptive pain, not neuropathic pain. Choice H is incorrect. The vital signs are not normal with visceral pain. They are often increased.

The nurse is caring for a client with an acute migraine headache. The nurse would anticipate a prescription for which medication? Select all that apply. A. Ketorolac B. Nitroglycerin C. Topiramate D. Dexamethasone E. Hydromorphone F. Acetaminophen-caffeine

Explanation Choices A, D, and F are correct. Treatment for an acute migraine headache (MH) involves abortive medications such as ketorolac (NSAID), dexamethasone (corticosteroid), and acetaminophen-caffeine. Depending on the severity of the MH, the provider takes a stepwise or aggressive approach to treatment. Choices B, C, and E are incorrect. Nitroglycerin would intensify the headache and would be unhelpful during an acute MH. Topiramate is an anticonvulsant and is a preventative treatment. This medication takes time to establish efficacy and is not used for acute migraines. Opioids (fentanyl, morphine, hydromorphone) are not recommended in managing an MH because they are likely to cause a paradoxical headache. Additional Info The treatment for an acute migraine headache aims to abort the headache and the associative symptoms such as nausea and vomiting. Commonly, a client may be prescribed an anti-emetic such as metoclopramide to assist with abating the symptoms. The below table reviews the treatment options for a migraine headache.

You are working in a Family Practice office. A patient comes into the office with right facial drooping. The physician makes a diagnosis of Bell's palsy. You know that the primary treatment for this disease is likely to include: Select all that apply. A. Surgery B. Prednisone C. Antibiotic D. Antivirals

Explanation Choices B and D are correct. Prednisone or another corticosteroid is likely to be prescribed. The anti-inflammatory action of these medications may help to reduce the swelling of the facial nerve and lessen the impingement that is causing the facial drooping. Antivirals are controversial, but some studies show that the combination of antivirals with corticosteroids may be helpful in patients with severe facial drooping. Both of these medications should be given as soon as possible after the symptoms start. Physical therapy to massage facial muscles can help to minimize permanent damage. Choice A is incorrect. Although surgery was once a treatment for Bell's palsy, it is no longer recommended due to the risk of permanent nerve injury and hearing loss. Choice C is incorrect. Antibiotics provide no relief for this condition since it is not caused by a bacteria that will respond to medicine. NCSBN Client Need Topic: Pharmacological and Parenteral Therapies, Sub-Topic: Expected Actions/Outcomes, Neurologic

The nurse is caring for a patient who is experiencing status epilepticus. Which of the following actions should be prioritized by the nurse? Select all that apply. A. Administer prescribed carbamazepine. B. Notify the rapid response team (RRT). C. Obtain a prescription for lorazepam. D. Loosen any restrictive clothing. E. Review the client's most recent phenytoin level.

Explanation Choices B, C, and D are correct. A client experiencing status epilepticus will require aggressive treatment as this is a persistent seizure that continues to recur despite treatment or a seizure that has lasted more than five minutes. The RRT should be notified as this is a medical emergency and requires evaluation by the RRT team. Obtaining a prescription for a parenteral benzodiazepine such as lorazepam is appropriate and should be completed by the nurse. Benzodiazepines are key in terminating a seizure. Central to caring for a client with a seizure is, placing them on their side and loosening any restrictive clothing. Choices A and E are incorrect. The client will likely require antiepileptic drugs to prevent future seizures; however, this is not the priority as carbamazepine is a maintenance drug used for seizure prevention. Additionally, reviewing drug levels that may assist in determining why a seizure may have occurred is not a priority during this medical emergency. The normal phenytoin level is 10-20 mcg/mL. Additional Info Status epilepticus is a medical emergency. This is when a seizure has lasted five or more minutes. Additionally, status epilepticus is defined as repeated seizure activity over the course of thirty minutes. During an acute seizure, the nurse should place the client on their side, loosen restrictive clothing, and anticipate a prescription for a parenteral benzodiazepine such as lorazepam. Status epilepticus requires more aggressive medical treatment, such as high-dose benzodiazepines or barbiturates. Last Updated - 20, May 2022

Which of the following symptoms are indicative of autonomic dysreflexia in a client who has experienced spinal cord injury? Select all that apply. A. Hypotension B. Sudden headache C. Flushed face D. Nasal congestion Submit Answer

Explanation Choices B, C, and D are correct. All of these answer choices are symptoms of autonomic dysreflexia. Autonomic dysreflexia (AD) is a condition in which the involuntary nervous system overreacts to external or bodily stimuli. It's also known as autonomic hyperreflexia. This reaction causes: A dangerous spike in blood pressure Bradycardia Constriction of your peripheral blood vessels Other changes in your body's autonomic functions The condition is most commonly seen in people with spinal cord injuries above the sixth thoracic vertebra, or T6. It may also affect people who have multiple sclerosis, Guillain-Barre Syndrome, and some head or brain injuries. AD can also be a side effect of medication or drug use. AD is a severe condition that's considered a medical emergency. It can be life-threatening and result in: Stroke Retinal hemorrhage Cardiac arrest Pulmonary edema The symptoms of AD may include: Anxiety and apprehension Nasal congestion High blood pressure with systolic readings often over 200 mmHg A pounding headache Flushing of the skin Profuse sweating, particularly on the forehead Lightheadedness Dizziness Confusion Dilated pupils Choice A is incorrect. Hypotension is not a symptom of autonomic dysreflexia. Instead, hypertension is indicative of autonomic dysreflexia. NCSBN Client Need Topic: Reduction of Risk Potential, Subtopic: Neurologic Last Updated - 27, Jan 2022

A male patient, one-day post-CVA, is showing signs of left-sided neglect. To begin the rehabilitation process, the nurse caring for this patient should add the following interventions to the patient's plan of care. Select all that apply. A. Sit on the unaffected side when interacting with the patient. B. Place the phone on the patient's affected side. C. Encourage the patient to touch the affected hand with the unaffected hand. D. Place a favorite object into the hand on the affected side.

Explanation Choices B, C, and D are correct. Rehabilitation should start as early as possible for the stroke victim with unilateral neglect. In this side effect of a stroke, the patient is not aware of one side of the body. In this scenario, the patient would ignore the left side of the body and might be unaware of anything happening to his left. The key to this question is the phrase "to begin the rehabilitation process." In this case, the purpose of rehab is to help the patient become aware of the side he is currently ignoring. These answer choices will all force the patient to acknowledge his left side. Sitting on the unaffected side will allow the patient to continue to ignore the left side; thus, the nurse should encourage visitors to interact with the patient from his left side. This may mean that the visitor will have to turn the patient's head physically to the left. Choice A is incorrect. NCSBN Client Need Topic: Reduction of Risk Potential, Sub-Topic: Therapeutic Procedures, Neurologic


Ensembles d'études connexes

Practice course test for Testing planning, management, and tool support

View Set

Chapter 18: Cardiovascular System II: The Blood Vessels

View Set

CALIFORNIA REAL ESTATE PRINCIPLES: CHAPTER 5: CONTRACTS. TERMS AND QUIZ.

View Set

Chapter 3: Getting the Job packet answers

View Set

Chapter 2 - Cognitive Neuroscience

View Set

Chaper 8 Quiz: Early Childhood Social and Emotional Development

View Set